You are on page 1of 42

Metabolic and Endocrine Management finding should nurse Hans recognize as an adverse drug

effect?
Endocrine Disorders 1 25 items
Dysuria
1. An agitated, confused female client arrives in the Leg cramps
emergency department. Her history includes type 1 diabetes Tachycardia
mellitus, hypertension, and angina pectoris. Assessment Blurred vision
reveals pallor, diaphoresis, headache, and intense hunger. A 7. A 67-year-old male client has been complaining of sleeping
stat blood glucose sample measures 42 mg/dl, and the client more, increased urination, anorexia, weakness, irritability,
is treated for an acute hypoglycemic reaction. After recovery, depression, and bone pain that interferes with her going
nurse Lily teaches the client to treat hypoglycemia by outdoors. Based on these assessment findings, nurse Richard
ingesting: would suspect which of the following disorders?

2 to 5 g of a simple carbohydrate. Diabetes mellitus


10 to 15 g of a simple carbohydrate. Diabetes insipidus
18 to 20 g of a simple carbohydrate. Hypoparathyroidism
25 to 30 g of a simple carbohydrate. Hyperparathyroidism
2. A female adult client with a history of chronic 8. When caring for a male client with diabetes insipidus,
hyperparathyroidism admits to being noncompliant. Based nurse Juliet expects to administer:
on initial assessment findings, nurse Julia formulates the
nursing diagnosis of Risk for injury. To complete the nursing vasopressin (Pitressin Synthetic).
diagnosis statement for this client, which “related-to” phrase furosemide (Lasix).
should the nurse add? regular insulin.
10% dextrose.
Related to bone demineralization resulting in pathologic 9. The nurse is aware that the following is the most common
fractures cause of hyperaldosteronism?
Related to exhaustion secondary to an accelerated metabolic
rate Excessive sodium intake
Related to edema and dry skin secondary to fluid infiltration A pituitary adenoma
into the interstitial spaces Deficient potassium intake
Related to tetany secondary to a decreased serum calcium An adrenal adenoma
level 10. A male client with type 1 diabetes mellitus has a highly
3. Nurse John is assigned to care for a postoperative male elevated glycosylated hemoglobin (Hb) test result. In
client who has diabetes mellitus. During the assessment discussing the result with the client, nurse Sharmaine would
interview, the client reports that he’s impotent and says he’s be most accurate in stating:
concerned about its effect on his marriage. In planning this
client’s care, the most appropriate intervention would be to: “The test needs to be repeated following a 12-hour fast.”
“It looks like you aren’t following the prescribed diabetic
Encourage the client to ask questions about personal diet.”
sexuality. “It tells us about your sugar control for the last 3 months.”
Provide time for privacy. “Your insulin regimen needs to be altered significantly.”
Provide support for the spouse or significant other. 11. Following a unilateral adrenalectomy, nurse Betty would
Suggest referral to a sex counselor or other appropriate assess for hyperkalemia shown by which of the following?
professional.
4. During a class on exercise for diabetic clients, a female Muscle weakness
client asks the nurse educator how often to exercise. The Tremors
nurse educator advises the clients to exercise how often to Diaphoresis
meet the goals of planned exercise? Constipation
12. Nurse Louie is developing a teaching plan for a male client
At least once a week diagnosed with diabetes insipidus. The nurse should include
At least three times a week information about which hormone lacking in clients with
At least five times a week diabetes insipidus?
Every day
5. Nurse Oliver should expect a client with hypothyroidism to antidiuretic hormone (ADH).
report which health concerns? thyroid-stimulating hormone (TSH).
follicle-stimulating hormone (FSH).
Increased appetite and weight loss luteinizing hormone (LH).
Puffiness of the face and hands 13. Early this morning, a female client had a subtotal
Nervousness and tremors thyroidectomy. During evening rounds, nurse Tina assesses
Thyroid gland swelling the client, who now has nausea, a temperature of 105° F
6. A female client with hypothyroidism (myxedema) is (40.5° C), tachycardia, and extreme restlessness. What is the
receiving levothyroxine (Synthroid), 25 mcg P.O. daily. Which most likely cause of these signs?
Diabetic ketoacidosis Consuming a low-carbohydrate, high-protein diet and
Thyroid crisis avoiding fasting.
Hypoglycemia 21. An incoherent female client with a history of
Tetany hypothyroidism is brought to the emergency department by
14. For a male client with hyperglycemia, which assessment the rescue squad. Physical and laboratory findings reveal
finding best supports a nursing diagnosis of Deficient fluid hypothermia, hypoventilation, respiratory acidosis,
volume? bradycardia, hypotension, and nonpitting edema of the face
and pretibial area. Knowing that these findings suggest
Cool, clammy skin severe hypothyroidism, nurse Libby prepares to take
Distended neck veins emergency action to prevent the potential complication of:
Increased urine osmolarity
Decreased serum sodium level Thyroid storm.
15. When assessing a male client with pheochromocytoma, a Cretinism.
tumor of the adrenal medulla that secretes excessive myxedema coma.
catecholamine, nurse April is most likely to detect: Hashimoto’s thyroiditis.
22. A male client with type 1 diabetes mellitus asks the nurse
a blood pressure of 130/70 mm Hg. about taking an oral antidiabetic agent. Nurse Jack explains
a blood glucose level of 130 mg/dl. that these medications are only effective if the client:
bradycardia.
a blood pressure of 176/88 mm Hg. prefers to take insulin orally.
16. A male client is admitted for treatment of the syndrome has type 2 diabetes.
of inappropriate antidiuretic hormone (SIADH). Which has type 1 diabetes.
nursing intervention is appropriate? is pregnant and has type 2 diabetes.
23. When caring for a female client with a history of
Infusing I.V. fluids rapidly as ordered hypoglycemia, nurse Ruby should avoid administering a drug
Encouraging increased oral intake that may potentiate hypoglycemia. Which drug fits this
Restricting fluids description?
Administering glucose-containing I.V. fluids as ordered
17. A female client has a serum calcium level of 7.2 mg/dl. sulfisoxazole (Gantrisin)
During the physical examination, nurse Noah expects to mexiletine (Mexitil)
assess: prednisone (Orasone)
lithium carbonate (Lithobid)
Trousseau’s sign. 24. After taking glipizide (Glucotrol) for 9 months, a male
Homans’ sign. client experiences secondary failure. Which of the following
Hegar’s sign. would the nurse expect the physician to do?
Goodell’s sign.
18. Which outcome indicates that treatment of a male client Initiate insulin therapy.
with diabetes insipidus has been effective? Switch the client to a different oral antidiabetic agent.
Prescribe an additional oral antidiabetic agent.
Fluid intake is less than 2,500 ml/day. Restrict carbohydrate intake to less than 30% of the total
Urine output measures more than 200 ml/hour. caloric intake.
Blood pressure is 90/50 mm Hg. 25. During preoperative teaching for a female client who will
The heart rate is 126 beats/minute. undergo subtotal thyroidectomy, the nurse should include
19. Jemma, who weighs 210 lb (95 kg) and has been which statement?
diagnosed with hyperglycemia tells the nurse that her
husband sleeps in another room because her snoring keeps “The head of your bed must remain flat for 24 hours after
him awake. The nurse notices that she has large hands and a surgery.”
hoarse voice. Which of the following would the nurse suspect “You should avoid deep breathing and coughing after
as a possible cause of the client’s hyperglycemia? surgery.”
“You won’t be able to swallow for the first day or two.”
Acromegaly “You must avoid hyperextending your neck after surgery.”
Type 1 diabetes mellitus
Hypothyroidism Answers and Rationales
Deficient growth hormone 1. Answer B. To reverse hypoglycemia, the American
20. Nurse Kate is providing dietary instructions to a male Diabetes Association recommends ingesting 10 to 15 g
client with hypoglycemia. To control hypoglycemic episodes, of a simple carbohydrate, such as three to five pieces
the nurse should recommend: of hard candy, two to three packets of sugar (4 to 6
tsp), or 4 oz of fruit juice. If necessary, this treatment
Increasing saturated fat intake and fasting in the afternoon. can be repeated in 15 minutes. Ingesting only 2 to 5 g
Increasing intake of vitamins B and D and taking iron of a simple carbohydrate may not raise the blood
supplements. glucose level sufficiently. Ingesting more than 15 g may
Eating a candy bar if light-headedness occurs. raise it above normal, causing hyperglycemia.
2. Answer A. Poorly controlled hyperparathyroidism may period. The test helps identify trends or practices that
cause an elevated serum calcium level. This, in turn, impair glycemic control, and it doesn’t require a fasting
may diminish calcium stores in the bone, causing bone period before blood is drawn. The nurse can’t conclude
demineralization and setting the stage for pathologic that the result occurs from poor dietary management
fractures and a risk for injury. Hyperparathyroidism or inadequate insulin coverage.
doesn’t accelerate the metabolic rate. A decreased 11. Answer A. Muscle weakness, bradycardia, nausea,
thyroid hormone level, not an increased parathyroid diarrhea, and paresthesia of the hands, feet, tongue,
hormone level, may cause edema and dry skin and face are findings associated with hyperkalemia,
secondary to fluid infiltration into the interstitial which is transient and occurs from transient
spaces. Hyperparathyroidism causes hypercalcemia, hypoaldosteronism when the adenoma is removed.
not hypocalcemia; therefore, it isn’t associated with Tremors, diaphoresis, and constipation aren’t seen in
tetany. hyperkalemia.
3. Answer D. The nurse should refer this client to a sex 12. Answer A. ADH is the hormone clients with diabetes
counselor or other professional. Making appropriate insipidus lack. The client’s TSH, FSH, and LH levels
referrals is a valid part of planning the client’s care. won’t be affected.
The nurse doesn’t normally provide sex counseling. 13. Answer B. Thyroid crisis usually occurs in the first 12
4. Answer B. Diabetic clients must exercise at least three hours after thyroidectomy and causes exaggerated
times a week to meet the goals of planned exercise — signs of hyperthyroidism, such as high fever,
lowering the blood glucose level, reducing or tachycardia, and extreme restlessness. Diabetic
maintaining the proper weight, increasing the serum ketoacidosis is more likely to produce polyuria,
high-density lipoprotein level, decreasing serum polydipsia, and polyphagia; hypoglycemia, to produce
triglyceride levels, reducing blood pressure, and weakness, tremors, profuse perspiration, and hunger.
minimizing stress. Exercising once a week wouldn’t Tetany typically causes uncontrollable muscle spasms,
achieve these goals. Exercising more than three times stridor, cyanosis, and possibly asphyxia.
a week, although beneficial, would exceed the 14. Answer C. In hyperglycemia, urine osmolarity (the
minimum requirement. measurement of dissolved particles in the urine)
5. Answer B. Hypothyroidism (myxedema) causes facial increases as glucose particles move into the urine. The
puffiness, extremity edema, and weight gain. Signs and client experiences glucosuria and polyuria, losing body
symptoms of hyperthyroidism (Graves’ disease) fluids and experiencing fluid volume deficit. Cool,
include an increased appetite, weight loss, clammy skin; distended neck veins; and a decreased
nervousness, tremors, and thyroid gland enlargement serum sodium level are signs of fluid volume excess,
(goiter). the opposite imbalance.
6. Answer C. Levothyroxine, a synthetic thyroid hormone, 15. Answer D. Pheochromocytoma, a tumor of the adrenal
is given to a client with hypothyroidism to simulate the medulla that secretes excessive catecholamine, causes
effects of thyroxine. Adverse effects of this agent hypertension, tachycardia, hyperglycemia,
include tachycardia. The other options aren’t hypermetabolism, and weight loss. It isn’t associated
associated with levothyroxine. with the other options.
7. Answer D. Hyperparathyroidism is most common in 16. Answer C. To reduce water retention in a client with
older women and is characterized by bone pain and the SIADH, the nurse should restrict fluids.
weakness from excess parathyroid hormone (PTH). Administering fluids by any route would further
Clients also exhibit hypercaliuria-causing polyuria. increase the client’s already heightened fluid load.
While clients with diabetes mellitus and diabetes 17. Answer A. This client’s serum calcium level indicates
insipidus also have polyuria, they don’t have bone pain hypocalcemia, an electrolyte imbalance that causes
and increased sleeping. Hypoparathyroidism is Trousseau’s sign (carpopedal spasm induced by
characterized by urinary frequency rather than inflating the blood pressure cuff above systolic
polyuria. pressure). Homans’ sign (pain on dorsiflexion of the
8. Answer A. Because diabetes insipidus results from foot) indicates deep vein thrombosis. Hegar’s sign
decreased antidiuretic hormone (vasopressin) (softening of the uterine isthmus) and Goodell’s sign
production, the nurse should expect to administer (cervical softening) are probable signs of pregnancy.
synthetic vasopressin for hormone replacement 18. Answer A. Diabetes insipidus is characterized by
therapy. Furosemide, a diuretic, is contraindicated polyuria (up to 8 L/day), constant thirst, and an
because a client with diabetes insipidus experiences unusually high oral intake of fluids. Treatment with the
polyuria. Insulin and dextrose are used to treat appropriate drug should decrease both oral fluid
diabetes mellitus and its complications, not diabetes intake and urine output. A urine output of 200 ml/hour
insipidus. indicates continuing polyuria. A blood pressure of
9. Answer D. An autonomous aldosterone-producing 90/50 mm Hg and a heart rate of 126 beats/minute
adenoma is the most common cause of indicate compensation for the continued fluid deficit,
hyperaldosteronism. Hyperplasia is the second most suggesting that treatment hasn’t been effective.
frequent cause. Aldosterone secretion is independent 19. Answer A. Acromegaly, which is caused by a pituitary
of sodium and potassium intake as well as of pituitary tumor that releases excessive growth hormone, is
stimulation. associated with hyperglycemia, hypertension,
10. Answer C. The glycosylated Hb test provides an diaphoresis, peripheral neuropathy, and joint pain.
objective measure of glycemic control over a 3-month Enlarged hands and feet are related to lateral bone
growth, which is seen in adults with this disorder. The 2. A male client with primary diabetes insipidus is ready for
accompanying soft tissue swelling causes hoarseness discharge on desmopressin (DDAVP). Which instruction
and often sleep apnea. Type 1 diabetes is usually seen should nurse Lina provide?
in children, and newly diagnosed persons are usually
very ill and thin. Hypothyroidism isn’t associated with “Administer desmopressin while the suspension is cold.”
hyperglycemia, nor is growth hormone deficiency. “Your condition isn’t chronic, so you won’t need to wear a
20. Answer D. To control hypoglycemic episodes, the nurse medical identification bracelet.”
should instruct the client to consume a low- “You may not be able to use desmopressin nasally if you have
carbohydrate, high-protein diet, avoid fasting, and nasal discharge or blockage.”
avoid simple sugars. Increasing saturated fat intake “You won’t need to monitor your fluid intake and output
and increasing vitamin supplementation wouldn’t help after you start taking desmopressin.”
control hypoglycemia. 3. Nurse Frank is aware that a positive Chvostek’s sign
21. Answer C. Severe hypothyroidism may result in indicate?
myxedema coma, in which a drastic drop in the
metabolic rate causes decreased vital signs, Hypocalcemia
hypoventilation (possibly leading to respiratory Hyponatremia
acidosis), and nonpitting edema. Thyroid storm is an Hypokalemia
acute complication of hyperthyroidism. Cretinism is a Hypermagnesemia
form of hypothyroidism that occurs in infants. 4. In a 29-year-old female client who is being successfully
Hashimoto’s thyroiditis is a common chronic treated for Cushing’s syndrome, nurse Lyzette would expect a
inflammatory disease of the thyroid gland in which decline in:
autoimmune factors play a prominent role.
22. Answer B. Oral antidiabetic agents are only effective in Serum glucose level.
adult clients with type 2 diabetes. Oral antidiabetic Hair loss.
agents aren’t effective in type 1 diabetes. Pregnant and Bone mineralization.
lactating women aren’t prescribed oral antidiabetic Menstrual flow.
agents because the effect on the fetus is uncertain. 5. A male client has recently undergone surgical removal of a
23. Answer A. Sulfisoxazole and other sulfonamides are pituitary tumor. Dr. Wong prescribes corticotropin (Acthar),
chemically related to oral antidiabetic agents and may 20 units I.M. q.i.d. as a replacement therapy. What is the
precipitate hypoglycemia. Mexiletine, an mechanism of action of corticotropin?
antiarrhythmic, is used to treat refractory ventricular
arrhythmias; it doesn’t cause hypoglycemia. It decreases cyclic adenosine monophosphate (cAMP)
Prednisone, a corticosteroid, is associated with production and affects the metabolic rate of target organs.
hyperglycemia. Lithium may cause transient It interacts with plasma membrane receptors to inhibit
hyperglycemia, not hypoglycemia. enzymatic actions.
24. Answer B. Many clients (25% to 60%) with secondary It interacts with plasma membrane receptors to produce
failure respond to a different oral antidiabetic agent. enzymatic actions that affect protein, fat, and carbohydrate
Therefore, it wouldn’t be appropriate to initiate insulin metabolism.
therapy at this time. However, if a new oral It regulates the threshold for water resorption in the kidneys.
antidiabetic agent is unsuccessful in keeping glucose 6. Capillary glucose monitoring is being performed every 4
levels at an acceptable level, insulin may be used in hours for a female client diagnosed with diabetic
addition to the antidiabetic agent. ketoacidosis. Insulin is administered using a scale of regular
25. Answer D. To prevent undue pressure on the surgical insulin according to glucose results. At 2 p.m., the client has a
incision after subtotal thyroidectomy, the nurse should capillary glucose level of 250 mg/dl for which he receives 8 U
advise the client to avoid hyperextending the neck. of regular insulin. Nurse Vince should expect the dose’s:
The client may elevate the head of the bed as desired
and should perform deep breathing and coughing to Onset to be at 2 p.m. and its peak to be at 3 p.m.
help prevent pneumonia. Subtotal thyroidectomy Onset to be at 2:15 p.m. and its peak to be at 3 p.m.
doesn’t affect swallowing. Onset to be at 2:30 p.m. and its peak to be at 4 p.m.
Onset to be at 4 p.m. and its peak to be at 6 p.m.
Endocrine Disorders 2 25 items 7. A female client with Cushing’s syndrome is admitted to the
medical-surgical unit. During the admission assessment,
nurse Tyzz notes that the client is agitated and irritable, has
1. Nurse Ronn is assessing a client with possible Cushing’s poor memory, reports loss of appetite, and appears
syndrome. In a client with Cushing’s syndrome, the nurse disheveled. These findings are consistent with which
would expect to find: problem?

Hypotension. Depression
Thick, coarse skin. Neuropathy
Deposits of adipose tissue in the trunk and dorsocervical Hypoglycemia
area. Hyperthyroidism
Weight gain in arms and legs. 8. Nurse Ruth is assessing a client after a thyroidectomy. The
assessment reveals muscle twitching and tingling, along with
numbness in the fingers, toes, and mouth area. The nurse necessitates a transphenoidal hypophysectomy. The evening
should suspect which complication? before the surgery, nurse Jacob reviews preoperative and
postoperative instructions given to the client earlier. Which
Tetany postoperative instruction should the nurse emphasize?
Hemorrhage
Thyroid storm “You must lie flat for 24 hours after surgery.”
Laryngeal nerve damage “You must avoid coughing, sneezing, and blowing your nose.”
9. After undergoing a subtotal thyroidectomy, a female client “You must restrict your fluid intake.”
develops hypothyroidism. Dr. Smith prescribes levothyroxine “You must report ringing in your ears immediately.”
(Levothroid), 25 mcg P.O. daily. For which condition is 16. Dr. Kennedy prescribes glipizide (Glucotrol), an oral
levothyroxine the preferred agent? antidiabetic agent, for a male client with type 2 diabetes
mellitus who has been having trouble controlling the blood
Primary hypothyroidism glucose level through diet and exercise. Which medication
Graves’ disease instruction should the nurse provide?
Thyrotoxicosis
Euthyroidism “Be sure to take glipizide 30 minutes before meals.”
10. Which of these signs suggests that a male client with the “Glipizide may cause a low serum sodium level, so make sure
syndrome of inappropriate antidiuretic hormone (SIADH) you have your sodium level checked monthly.”
secretion is experiencing complications? “You won’t need to check your blood glucose level after you
start taking glipizide.”
Tetanic contractions “Take glipizide after a meal to prevent heartburn.”
Neck vein distention 17. For a diabetic male client with a foot ulcer, the physician
Weight loss orders bed rest, a wet-to-dry dressing change every shift, and
Polyuria blood glucose monitoring before meals and bedtime. Why
11. A female client with a history of pheochromocytoma is are wet-to-dry dressings used for this client?
admitted to the hospital in an acute hypertensive crisis. To
reverse hypertensive crisis caused by pheochromocytoma, They contain exudate and provide a moist wound
nurse Lyka expects to administer: environment.
They protect the wound from mechanical trauma and
phentolamine (Regitine). promote healing.
methyldopa (Aldomet). They debride the wound and promote healing by secondary
mannitol (Osmitrol). intention.
felodipine (Plendil). They prevent the entrance of microorganisms and minimize
12. A male client with a history of hypertension is diagnosed wound discomfort.
with primary hyperaldosteronism. This diagnosis indicates 18. When instructing the female client diagnosed with
that the client’s hypertension is caused by excessive hormone hyperparathyroidism about diet, nurse Gina should stress the
secretion from which of the following glands? importance of which of the following?

Adrenal cortex Restricting fluids


Pancreas Restricting sodium
Adrenal medulla Forcing fluids
Parathyroid Restricting potassium
13. Nurse Troy is aware that the most appropriate for a client 19. Which nursing diagnosis takes highest priority for a
with Addison’s disease? female client with hyperthyroidism?

Risk for infection Risk for imbalanced nutrition: More than body requirements
Excessive fluid volume related to thyroid hormone excess
Urinary retention Risk for impaired skin integrity related to edema, skin
Hypothermia fragility, and poor wound healing
14. Acarbose (Precose), an alpha-glucosidase inhibitor, is Body image disturbance related to weight gain and edema
prescribed for a female client with type 2 diabetes mellitus. Imbalanced nutrition: Less than body requirements related to
During discharge planning, nurse Pauleen would be aware of thyroid hormone excess
the client’s need for additional teaching when the client 20. A male client with a tentative diagnosis of hyperosmolar
states: hyperglycemic nonketotic syndrome (HHNS) has a history of
type 2 diabetes that is being controlled with an oral diabetic
“If I have hypoglycemia, I should eat some sugar, not agent, tolazamide (Tolinase). Which of the following is the
dextrose.” most important laboratory test for confirming this disorder?
“The drug makes my pancreas release more insulin.”
“I should never take insulin while I’m taking this drug.” Serum potassium level
“It’s best if I take the drug with the first bite of a meal.” Serum sodium level
15. A female client whose physical findings suggest a Arterial blood gas (ABG) values
hyperpituitary condition undergoes an extensive diagnostic Serum osmolarity
workup. Test results reveal a pituitary tumor, which
21. A male client has just been diagnosed with type 1 intake and output and receive adequate fluid
diabetes mellitus. When teaching the client and family how replacement.
diet and exercise affect insulin requirements, Nurse Joy 3. Answer A. Chvostek’s sign is elicited by tapping the
should include which guideline? client’s face lightly over the facial nerve, just below the
temple. If the client’s facial muscles twitch, it indicates
“You’ll need more insulin when you exercise or increase your hypocalcemia. Hyponatremia is indicated by weight
food intake.” loss, abdominal cramping, muscle weakness,
“You’ll need less insulin when you exercise or reduce your headache, and postural hypotension. Hypokalemia
food intake.” causes paralytic ileus and muscle weakness. Clients
“You’ll need less insulin when you increase your food intake.” with hypermagnesemia exhibit a loss of deep tendon
“You’ll need more insulin when you exercise or decrease your reflexes, coma, or cardiac arrest.
food intake.” 4. Answer A. Hyperglycemia, which develops from
22. Nurse Noemi administers glucagon to her diabetic client, glucocorticoid excess, is a manifestation of Cushing’s
then monitors the client for adverse drug reactions and syndrome. With successful treatment of the disorder,
interactions. Which type of drug interacts adversely with serum glucose levels decline. Hirsutism is common in
glucagon? Cushing’s syndrome; therefore, with successful
treatment, abnormal hair growth also declines.
Oral anticoagulants Osteoporosis occurs in Cushing’s syndrome; therefore,
Anabolic steroids with successful treatment, bone mineralization
Beta-adrenergic blockers increases. Amenorrhea develops in Cushing’s
Thiazide diuretics syndrome. With successful treatment, the client
23. Which instruction about insulin administration should experiences a return of menstrual flow, not a decline
nurse Kate give to a client? in it.
5. Answer C. Corticotropin interacts with plasma
“Always follow the same order when drawing the different membrane receptors to produce enzymatic actions
insulins into the syringe.” that affect protein, fat, and carbohydrate metabolism.
“Shake the vials before withdrawing the insulin.” It doesn’t decrease cAMP production. The posterior
“Store unopened vials of insulin in the freezer at pituitary hormone, antidiuretic hormone, regulates the
temperatures well below freezing.” threshold for water resorption in the kidneys.
“Discard the intermediate-acting insulin if it appears cloudy.” 6. Answer C. Regular insulin, which is a short-acting
24. Nurse Perry is caring for a female client with type 1 insulin, has an onset of 15 to 30 minutes and a peak of
diabetes mellitus who exhibits confusion, light-headedness, 2 to 4 hours. Because the nurse gave the insulin at 2
and aberrant behavior. The client is still conscious. The nurse p.m., the expected onset would be from 2:15 p.m. to
should first administer: 2:30 p.m. and the peak from 4 p.m. to 6 p.m.
7. Answer A. Agitation, irritability, poor memory, loss of
I.M. or subcutaneous glucagon. appetite, and neglect of one’s appearance may signal
I.V. bolus of dextrose 50%. depression, which is common in clients with Cushing’s
15 to 20 g of a fast-acting carbohydrate such as orange juice. syndrome. Neuropathy affects clients with diabetes
10 U of fast-acting insulin. mellitus — not Cushing’s syndrome. Although
25. For the first 72 hours after thyroidectomy surgery, nurse hypoglycemia can cause irritability, it also produces
Jamie would assess the female client for Chvostek’s sign and increased appetite, rather than loss of appetite.
Trousseau’s sign because they indicate which of the Hyperthyroidism typically causes such signs as goiter,
following? nervousness, heat intolerance, and weight loss despite
increased appetite.
Hypocalcemia 8. Answer A. Tetany may result if the parathyroid glands
Hypercalcemia are excised or damaged during thyroid surgery.
Hypokalemia Hemorrhage is a potential complication after thyroid
Hyperkalemia surgery but is characterized by tachycardia,
hypotension, frequent swallowing, feelings of fullness
Answers and Rationales at the incision site, choking, and bleeding. Thyroid
1. Answer C. Because of changes in fat distribution, storm is another term for severe hyperthyroidism —
adipose tissue accumulates in the trunk, face not a complication of thyroidectomy. Laryngeal nerve
(moonface), and dorsocervical areas (buffalo hump). damage may occur postoperatively, but its signs
Hypertension is caused by fluid retention. Skin include a hoarse voice and, possibly, acute airway
becomes thin and bruises easily because of a loss of obstruction.
collagen. Muscle wasting causes muscle atrophy and 9. Answer A. Levothyroxine is the preferred agent to
thin extremities. treat primary hypothyroidism and cretinism, although
2. Answer C. Desmopressin may not be absorbed if the it also may be used to treat secondary hypothyroidism.
intranasal route is compromised. Although diabetes It is contraindicated in Graves’ disease and
insipidus is treatable, the client should wear medical thyrotoxicosis because these conditions are forms of
identification and carry medication at all times to alert hyperthyroidism. Euthyroidism, a term used to
medical personnel in an emergency and ensure proper describe normal thyroid function, wouldn’t require any
treatment. The client must continue to monitor fluid thyroid preparation.
10. Answer B. SIADH secretion causes antidiuretic measurement. The client must continue to monitor the
hormone overproduction, which leads to fluid blood glucose level during glipizide therapy.
retention. Severe SIADH can cause such complications 17. Answer C. For this client, wet-to-dry dressings are most
as vascular fluid overload, signaled by neck vein appropriate because they clean the foot ulcer by
distention. This syndrome isn’t associated with tetanic debriding exudate and necrotic tissue, thus promoting
contractions. It may cause weight gain and fluid healing by secondary intention. Moist, transparent
retention (secondary to oliguria). dressings contain exudate and provide a moist wound
11. Answer A. Pheochromocytoma causes excessive environment. Hydrocolloid dressings prevent the
production of epinephrine and norepinephrine, natural entrance of microorganisms and minimize wound
catecholamines that raise the blood pressure. discomfort. Dry sterile dressings protect the wound
Phentolamine, an alpha-adrenergic blocking agent from mechanical trauma and promote healing.
given by I.V. bolus or drip, antagonizes the body’s 18. Answer C. The client should be encouraged to force
response to circulating epinephrine and fluids to prevent renal calculi formation. Sodium
norepinephrine, reducing blood pressure quickly and should be encouraged to replace losses in urine.
effectively. Although methyldopa is an Restricting potassium isn’t necessary in
antihypertensive agent available in parenteral form, it hyperparathyroidism.
isn’t effective in treating hypertensive emergencies. 19. Answer D. In the client with hyperthyroidism,
Mannitol, a diuretic, isn’t used to treat hypertensive excessive thyroid hormone production leads to
emergencies. Felodipine, an antihypertensive agent, is hypermetabolism and increased nutrient metabolism.
available only in extended-release tablets and These conditions may result in a negative nitrogen
therefore doesn’t reduce blood pressure quickly balance, increased protein synthesis and breakdown,
enough to correct hypertensive crisis. decreased glucose tolerance, and fat mobilization and
12. Answer A. Excessive secretion of aldosterone in the depletion. This puts the client at risk for marked
adrenal cortex is responsible for the client’s nutrient and calorie deficiency, making Imbalanced
hypertension. This hormone acts on the renal tubule, nutrition: Less than body requirements the most
where it promotes reabsorption of sodium and important nursing diagnosis. Options B and C may be
excretion of potassium and hydrogen ions. The appropriate for a client with hypothyroidism, which
pancreas mainly secretes hormones involved in fuel slows the metabolic rate.
metabolism. The adrenal medulla secretes the 20. Answer D. Serum osmolarity is the most important test
catecholamines — epinephrine and norepinephrine. for confirming HHNS; it’s also used to guide treatment
The parathyroids secrete parathyroid hormone. strategies and determine evaluation criteria. A client
13. Answer A. Addison’s disease decreases the production with HHNS typically has a serum osmolarity of more
of all adrenal hormones, compromising the body’s than 350 mOsm/L. Serum potassium, serum sodium,
normal stress response and increasing the risk of and ABG values are also measured, but they aren’t as
infection. Other appropriate nursing diagnoses for a important as serum osmolarity for confirming a
client with Addison’s disease include Deficient fluid diagnosis of HHNS. A client with HHNS typically has
volume and Hyperthermia. Urinary retention isn’t hypernatremia and osmotic diuresis. ABG values reveal
appropriate because Addison’s disease causes acidosis, and the potassium level is variable.
polyuria. 21. Answer B. Exercise, reduced food intake,
14. Answer A. Acarbose delays glucose absorption, so the hypothyroidism, and certain medications decrease the
client should take an oral form of dextrose rather than insulin requirements. Growth, pregnancy, greater food
a product containing table sugar when treating intake, stress, surgery, infection, illness, increased
hypoglycemia. The alpha-glucosidase inhibitors work insulin antibodies, and certain medications increase
by delaying the carbohydrate digestion and glucose the insulin requirements.
absorption. It’s safe to be on a regimen that includes 22. Answer A. As a normal body protein, glucagon only
insulin and an alpha-glucosidase inhibitor. The client interacts adversely with oral anticoagulants, increasing
should take the drug at the start of a meal, not 30 the anticoagulant effects. It doesn’t interact adversely
minutes to an hour before. with anabolic steroids, beta-adrenergic blockers, or
15. Answer B. After a transsphenoidal hypophysectomy, thiazide diuretics.
the client must refrain from coughing, sneezing, and 23. Answer A. The client should be instructed always to
blowing the nose for several days to avoid disturbing follow the same order when drawing the different
the surgical graft used to close the wound. The head of insulins into the syringe. Insulin should never be
the bed must be elevated, not kept flat, to prevent shaken because the resulting froth prevents
tension or pressure on the suture line. Within 24 hours withdrawal of an accurate dose and may damage the
after a hypophysectomy, transient diabetes insipidus insulin protein molecules. Insulin also should never be
commonly occurs; this calls for increased, not frozen because the insulin protein molecules may be
restricted, fluid intake. Visual, not auditory, changes damaged. Intermediate-acting insulin is normally
are a potential complication of hypophysectomy. cloudy.
16. Answer A. The client should take glipizide twice a day, 24. Answer C. This client is having a hypoglycemic episode.
30 minutes before a meal, because food decreases its Because the client is conscious, the nurse should first
absorption. The drug doesn’t cause hyponatremia and administer a fast-acting carbohydrate, such as orange
therefore doesn’t necessitate monthly serum sodium juice, hard candy, or honey. If the client has lost
consciousness, the nurse should administer either I.M.
or subcutaneous glucagon or an I.V. bolus of dextrose 6. You assess a patient with Cushing’s disease. For which
50%. The nurse shouldn’t administer insulin to a client finding will you notify the physician immediately?
who’s hypoglycemic; this action will further
compromise the client’s condition. Purple striae present on abdomen and thighs
25. Answer A. The client who has undergone a Weight gain of 1 pound since the previous day
thyroidectomy is at risk for developing hypocalcemia +1 dependent edema in ankles and calves
from inadvertent removal or damage to the Crackles bilaterally in lower lobes of lungs
parathyroid gland. The client with hypocalcemia will 7. The patient with pheochromocytoma had surgery to
exhibit a positive Chvostek’s sign (facial muscle remove his adrenal glands. Which nursing intervention
contraction when the facial nerve in front of the ear is should you delegate to the nursing assistant?
tapped) and a positive Trousseau’s sign (carpal spasm
when a blood pressure cuff is inflated for a few Add strategies to provide a calm and restful environment
minutes). These signs aren’t present with post-operatively to the care plan.
hypercalcemia, hypokalemia, or hyperkalemia. Warm the patient to avoid smoking and drinking caffeinated
beverages
Endocrine Problems 22 items Monitor the patient’s skin and mucous membranes for signs
1. A patient is admitted to the medical unit with possible of adequate hydration.
Graves’ disease (hyperthyroidism). Which assessment finding Monitor lying and standing blood pressure every 4 hours with
supports this diagnosis? cuff placed on same arm
8. For the patient with pheochromocytoma, what physical
Periorbital edema assessment technique should you instruct the LPN/LVN to
Bradycardia avoid?
Exophthalmos
Hoarse voice __________________________________________________
2. Which change in vital signs would you instruct a nursing ______
assistant to report immediately for a patient with 9. The patient with adrenal insufficiency is to be discharged
hyperthyroidism? taking prednisone 10 mg orally each day. What will you be
sure to teach the patient?
Increased and rapid heart rate
Decrease systolic blood pressure Report excessive weigh gain or swelling to the physician.
Increased respiratory rate Rapid changes of position may cause hypotension.
Decreased oral temperature A diet with foods high in potassium may be beneficial.
3. For the patient with hyperthyroidism, what intervention Signs of hypoglycemia may occur while taking this drug.
should you delegate to the experienced certified nursing 10.You are caring for a patient who is post-hypophysectomy
assistant? for hyperpituitarism. Which post-operative finding requires
immediate intervention?
Instruct the patient to report palpitations, dyspnea, vertigo,
pr chest pain. Presence of glucose in nasal drainage
Check the apical pulse, blood pressure, and temperature Nasal packing present in nares
every 4 hours. Urine output of 40 – 50 mL per hour
Draw blood for thyroid-stimulating hormone, T3, and T4 Patient complaints of thirst
levels. 11.Which patient’s nursing care would be most appropriate
Explain the side effects of propylthiouracil (PTU) to the for the charge nurse to assign to the LPN, under the
patient. supervision of the RN team leader?
4. As the shift begins, you are assigned these patients. Which
patient should you assess first? A 51-year-old patient with bilateral adrenalectomy just
returned from the post-anesthesia care unit
A 38-year-old patient with Graves’ disease and a heart rate of An 83-year-old patient with type 2 diabetes and chronic
94/minute obstructive pulmonary disease
A 63-year-old patient with type 2 diabetes and fingerstick A 38-year-old patient with myocardial infarction who is
glucose of 137 mg/dL preparing for discharge
A 58-year-old patient with hypothyroidism and heart rate of A 72-year-old patient admitted from long-term care with
48/minute mental status changes
A 49-year-old patient with Cushing’s disease and +1 12.You are providing care for a patient who underwent
dependent edema thyroidectomy 2 days ago. Which laboratory value requires
5. A patient is hospitalized with adrenocortical insufficiency. close monitoring?
Which nursing activity should you delegate to the nursing
assistant? Calcium
Sodium
Remind patient to change positions slowly. Potassium
Check the patient for muscle weakness White blood cells
Teach the patient how to collect 24-hour urine
Plan nursing interventions to promote fluid balance
13.You are preparing to review a teaching plan for a patient Hammertoe of the left second metatarsophalangeal joint
with type 2 diabetes. What will you check to determine the Rapid respiratory rate with deep inspirations
patient’s level of compliance with his diabetic regimen? Numbness and tingling bilaterally in the feet and hands
Decreased sensitivity and swelling of the abdomen
Patient’s fasting glucose level 20.You are caring for a diabetic patient who is developing
Patient’s oral glucose tolerance test results DKA. Which delegated task is most appropriate?
Patient’s glycosylated hemoglobin assay
Patient’s fingerstick glucose check for 24 hours Ask the unit clerk to page the physician to come to the unit.
14.The patient has newly diagnosed type 2 diabetes. Which Ask the LPN/LVN to administer IV insulin according to the
task should you delegate to the nursing assistant? sliding scale.
Ask the nursing assistant to check the patient’s level of
Arrange consult with the dietician for patient. consciousness.
Verify patient’s insulin injection technique. Ask the nursing assistant to get the patient a cup of orange
Teach patient to use glucometer for monitoring glucose at juice.
home. 21.A diabetic patient presents with hot and dry skin, rapid
Remind patient to check glucose level prior to each meal. and deep respirations, and a fruity odor to his breath. As
15.A nursing diagnosis for the newly diagnosed diabetic charge nurse, you observe the new graduate RN
patient is risk for Injury related to sensory alterations. Which accomplishing all these patient tasks. Which one requires
key points should you include in the teaching plan for this that you intervene immediately?
patient? (Choose all that apply).
The RN checks the patient’s fingerstick glucose.
Clean and inspect your feet every day. The RN encourages the patient to drink orange juice.
Be sure that your shoes fit properly. The RN checks the patient’s order for sliding scale insulin.
Nylon socks are best to prevent friction between toes and The RN assess the patient’s vital signs every 15 minutes
shoes. 22.You are preparing a 24-year-old patient with diabetes
Report any non-healing skin breaks to your doctor. insipidus (DI) for discharge from the hospital. Which
16.The diabetic patient has all of these assessment bindings. statement indicates that the patient needs additional
Which will you instruct the LPN/LVN to report immediately? teaching?

Fingerstick glucose of 185 mg/dL “I will drink fluids equal to the amount of my urine output.”
Numbness and tingling in both feet “I will weigh myself every day using the same scale.”
Profuse perspiration “I will wear my medical alert bracelet at all times.”
Bunion on left great toe “I will gradually wean myself off the vasopressin.”
17.The plan of care for the diabetic patient includes all of the
following interventions. Which intervention could you Answers and Rationales
delegate to the nursing assistant? 1. ANSWER C – Exophthalmos (abnormal protrusion of
the eye) is characteristic of patients with
Check to make sure that the patient’s bath water is not too hyperthyroidism due to Graves’ disease. Periorbital
hot. edema, bradycardia, and hoarse voice are all
Discuss community resources for diabetic outpatient care. characteristics of patients with hypothyroidism. Focus:
Instruct the patient to perform daily foot inspections. Prioritization
Check the patient’s technique for drawing insulin into a 2. ANSWER A – The cardiac problems associated with
syringe. hyperthyroidism include tachycardia, increased systolic
18.You are precepting a nurse who has recently graduated blood pressure, and decreased diastolic blood
and passes the NCLEX examination. The new nurse has been pressure. Patients with hyperthyroidism also may have
on the unit for only 2 days. Which patient should you assign increased body temperature related to increased
to the new nurse? metabolic rate. Focus: Delegation/supervision
3. ANSWER B – Monitoring and recording vital signs are
A 68-year-old diabetic who is experiencing signs of within the education scope of nursing assistants. An
hyperglycemia including rapid, deep breathing and mental experienced nursing assistant should have been taught
status changes how to monitor the apical pulse. However, the nurse
A 58-year-old diabetic with peripheral neuropathy and should observe the nursing assistant to be sure that
cellulitis of the left ankle. she has mastered this skill. Instructing and teaching
A 49-year-old diabetic who has just returned from post- patients, as well as performing venipuncture for
anesthesia care unit (PACU) after a below-the-knee laboratory samples, are more suited to the educational
amputation (BKA) scope of licensed nurses. In some facilities, an
A 72-year-old diabetic with diabetic ketoacidosis (DKA) on an experienced nursing assistant may perform
IV insulin drip venipuncture, but only after special training. Focus:
19.In the emergency department, during initial assessment of Delegation/supervision
a new admission with diabetes, you discover all of the 4. ANSWER C – Although patients with hypothyroidism
following. Which information should you immediately report often have cardiac problems that include bradycardia,
to the physician? a heart rate of 48/minute may have significant
implications for cardiac output and hemodynamic
stability. Patients with Graves’ disease usually have a balance. The nurse should be attentive to all patient
rapid heart rate, but 94/minute is within limits. The laboratory values, but calcium and phosphorus are
diabetic patient may need sliding scale insulin. This is important to monitor after thyroidectomy. Focus:
important but not urgent. Patients with Cushing’s Prioritization
disease frequently have dependent edema. Focus: 13. ANSWER C – The higher the blood glucose level is over
Prioritization time, the more elevated the glycosylated hemoglobin
5. ANSWER A – Patients with hypofunction of the adrenal becomes. Glycosylated hemoglobin is a good indicator
glands often have hypotension and should be of average blood glucose level over the previous 120
instructed to change positions slowly. Once a patient days. Fasting glucose and oral glucose tolerance tests
has been instructed, it is appropriate for the nursing are important diagnostic tests. Fingerstick blood
assistant to remind the patient of those instructions. glucose monitoring provides information that allows
Assessing, teaching, and planning nursing care require for adjustment of patients’ therapeutic regimen.
more education and should be done by licensed Focus: Prioritization
nurses. Focus: Delegation/supervision 14. ANSWER D – The nursing assistant’s role includes
6. ANSWER D – The presence of crackles in the patient’s reminding patients about interventions that are
lungs indicate excess fluid volume doe to excess water already part of the plan of care. Arranging for a diet
and sodium reabsorption and may be a symptom of consult is appropriate to delegate to the unit clerk.
pulmonary edema, which must be treated rapidly. Teaching and assessing require additional education
Striae (stretch marks), weight gain, and dependent and should be completed by licensed nurses. Focus:
edema are common findings in patients Cushing’s Delegation/supervision, assignment
disease. These findings should be monitored, but are 15. ANSWER A, B & E – Sensory alterations are the major
not urgent. Focus: Prioritization cause of foot complications in diabetic patients, and
7. ANSWER D – Monitoring vital signs is within the patients should be taught to examine their feet on a
educational scope of the nursing assistant. The nurse daily basis. Properly fitted shoes protect the patient
should be sure to instruct the nursing assistant that from foot complications. Broken skin increases the risk
blood pressure measurements are to be done with the of infection. Cotton socks are recommended to absorb
cuff on the same arm. Revising the care plan and moisture. Patients, family, or health care providers
instructing and assessing patients are beyond the may trim toenails. Focus: Prioritization
scope of nursing assistants and fall within the purview 16. ANSWER C – Profuse perspiration is a symptom of
of licensed nurses. Focus: Delegation/supervision hypoglycemia, a complication of diabetes that needs
8. ANSWER: PALPATION OF THE ABDOMEN – Palpating urgent treatment. A glucose level of 185 will need
the abdomen can cause sudden release of coverage with sliding-scale insulin, but this is not
catecholamines and severe hypertension. Focus: urgent. Numbness, tingling, and bunions are related to
Delegation/supervision the chronic nature of diabetes and are not urgent.
9. ANSWER A – Rapid weight gain and edema are signs of Focus: Prioritization
excessive drug therapy, and the dose of the drug needs 17. ANSWER A – Checking the bath water temperature is
to be adjusted. Hypertension, hyperkalemia, and part of assisting with activities of daily living and is
hyperglycemia are common in patients with adrenal within the educational scope of the nursing assistant.
hypofunction. Focus: Prioritization Discussion of community resources and teaching and
10. ANSWER A – The presence of glucose in nasal drainage assessing require a higher level of education and are
indicates that the fluid is CSF (cerebrospinal fluid) and appropriate to the scope of practice of licensed nurses.
suggests a CSF leak. Packing is normally inserted in the Focus: Delegation
nares after the surgical incision is closed. Forty to 50 18. ANSWER B – The new nurse is still orienting to the
mL per hour is adequate urine output and patients unit. Appropriate patient assignments at this time
may experience thirst post-operatively. When patients include those who are stable and not complex. Focus:
are thirsty, nursing staff should encourage fluid intake. Assignment
Thirst may be a sign of hypokalemia. The nurse should 19. ANSWER B – Rapid, deep respirations (Kussmaul) are
assess the patient’s thirst and check the patient’s symptomatic of DKA. Hammertoe, as well as numbness
potassium level. This is not as urgent as the CSF leak. and tingling, are chronic complications associated with
Focus: Prioritization diabetes. Decreased sensitivity and swelling
11. ANSWER B – The 83-year-old patient has no (lipohypertrophy) occur at a site of repeated insulin
complicating factors at the moment. Providing care for injections, and treatment involves teaching the patient
stable and uncomplicated patients is within the LPN’s to rotate injection sites. Focus: Prioritization
educational preparation and scope of practice, with 20. ANSWER A – The nurse should not leave the patient.
the care always being provided under the supervision The scope of the unit clerk’s job includes calling and
and direction of the RN. The RN should assess the paging physicians. LPN/LVNs generally do not
newly post-operative patient and the new admission. administer IV push medication. IV fluid administration
The patient who is preparing for discharge after MI is not within the scope of nursing assistants. Patients
may need some complex teaching. Focus: with DKA already have a high glucose level and do not
Delegation/supervision, assignment need orange juice. Focus: Delegation/supervision
12. ANSWER A – The parathyroid glands are located on the 21. ANSWER B – The signs and symptoms the patient is
back of the thyroid gland. The parathyroids are exhibiting are consistent with hyperglycemia. The RN
important in maintaining calcium and phosphorus should not give the patient additional glucose. All of
the other interventions are appropriate for this
patient. The RN should also notify the physician at this SV/TPR
time. Focus: Prioritization CO/TPR
22. ANSWER D – The patient with permanent DI requires SV/EDV
life-long vasopressin therapy. All of the other CO/EDV
statements are appropriate to the home care of this 10. Which of the following is not a bradykinin effect?
patient. Focus: Prioritization
Elevated capillary permeability
Endocrine System 20 items Elevated pain levels
1. Which of the following hormones causes increased atrial Elevated vasodilation response
pressure and decreases sodium reabsorption in the kidneys? Elevated prothrombin secretion
11. PAH is secreted in which of the following locations?
Atrial natriuretic peptide
PTH Distal tubule
Aldosterone Loop of Henle
Vasopressin Collecting tubule
2. Angiotensin I is changed by which of the following into Proximal tubule
Angiotensin II? 12. Which of the following is not an anterior pituitary gland
secretion?
ACE
AVT TSH
Pepsin GH
Adenosine Vasopressin
3. Which of the following is not a cause of peripheral edema? Prolactin
13. Thyroid Hormone T3 does not have which of the
Increased capillary permeability following functions?
Reduced levels of plasma proteins
Heart failure Stimulate bone development and growth
Decreased capillary output Create beta-adrenergic responses
4. Which of the following during an electrocardiogram is Cause brain development
associated with hypokalemia? Decrease calcium re-absorption
14. Hypercalemia has not been linked with which of the
QRS complex following?
U wave
PR segment Paget’s disease
ST segment Aldosterone
5. Which of the following is not generally associated with a Sarcoidosis
2nd degree (Mobitz Type 1) AV block? Malignancy
15. Which of the following does not require the pre-cursor
Usually asymptomatic progesterone?
Nonsequential (P wave then QRS complex)
Increased PR segment/interval Cortisol
70% Fatal Testosterone
6. An S3 heart sound is often associated with? ACTH
Aldosterone
CHF 16. Which of the following is the source cell for the secretion
COPD Pepsinogen?
Atrial fib.
Ventricular fib. Chief cell
7. Mean arterial pressure is the product of: Plasma cell
G cell
TPR x SV Parietal cell
TPR x CO 17. Which of the following is the primary activator of
CO/SV zymogen secretion?
SV/EDV
8. During phase 3 of the myocardial action potential which Somatostatin
ion is moving the most? Secretin
Acetylcholine
K+ Gastrin
Ca+ 18. Which of the following is not a function of Angiotensin II?
Na+
Ca2+ Causes release of aldosterone
9. An ejection fraction can be calculated as: Causes vasodilation
Causes increased posterior pituitary activation FALSE
Elevates blood pressure 6. Which of the following is not true about Type I DM?
19. Which of the following is not a function of Progesterone?
May be linked to autoimmunity
Causes increased body temperature. Onset usually prior to age 20
Causes some smooth muscle relaxation. Beta islet cells destroyed
Causes increased spiral artery growth Does not require insulin injections
Causes activation of FSH 7. Which of the following is not true about Type II DM?
20. Which of the following is not a function of Estrogen?
Considered adult onset diabetes
Causes breast growth. Cause unknown may be due to genetics
Causes inhibition of FSH Require insulin 80% of cases
Increased follicle development May take a drug that sensitize cells or increase insulin release
Decreased overall transport proteins 8. Which of the following is not an effect of diabetes?

Answers Small vessel occlusion


1. A. Atrial natriuretic peptide Necrosis of extremities
2. A. ACE Ketone Body production
3. D. Decreased capillary output Decreased fat metabolism
4. B. U wave 9. Which of the following is not an indicator of a
5. D. 70% Fatal hypoglycemic condition?
6. A. CHF
7. B. TPR x CO Fatigue
8. A. K+ Poor appetite
9. C. SV/EDV Tachycardia
10. D. Elevated prothrombin secretion Confusion
11. D. Proximal tubule 10. Which of the following is not an adverse effect of oral
12. C. Vasopressin hypoglycemics?
13. D. Decrease calcium re-absorption
14. B. Aldosterone Hypoglycemia
15. C. ACTH Headache
16. A. Chief cell Rashes
17. C. Acetylcholine Projectile vomiting
18. B. Causes vasodilation 11. Which of the following is not an adverse effect of
19. D. Causes activation of FSH glucagon?
20. D. Decreased overall transport proteins
Allergic reaction
Pancreatic Organ 20 items Vomiting
1. Insulin inhibits the release of _______. Nausea
Fever
Glucagon 12. Which of the following drugs may be given as an
ADH immunosuppressant soon after onset of Type I Diabetes?
Beta cells
Somatostatin Torsemide
2. Which of the following is caused by insulin release? Cyclosporine
Clofibrate
Increased breakdown of fats Ceftriaxone
Increase breakdown of proteins 13. Which of the following is not considered an endocrine
Decreased blood sugar hormone?
Causes glucose to be phosphorylated in kidney
3. Glucagon causes increased blood sugar and causes slow Renin
breakdown of glycogen in the liver. Insulin
Glucagon
TRUE Somatostatin
FALSE 14. What type of cells secrete glucagon?
4. As blood glucose decreases glucagon is inhibited.
Beta cells
TRUE Alpha cells
FALSE Plasma cells
5. Glucagon increases blood levels of glucose by causing liver Acinar cells
to breakdown glycogen. 15. What type of cells secrete insulin?

TRUE Beta cells


Alpha cells 1) A patient suspected of having acromegaly has an elevated
Plasma cells plasma growth hormone level. In acromegaly, the nurse
Acinar cells would also expect the patient’s diagnostic results to include
16. Which of the following would not be considered an acute
effect of diabetes mellitus? hyperinsulinemia
a plasma glucose of less than 70
Polyuria decreased growth hormone levels with an oral glucose
Weight gain challenge test
Polydipsia a serum sometomedin C (insulin-like growth-factor) of more
Polyphagia than 300
17. Which of the following is not an accurate test for 2) During assessment of the patient with acromegaly, the
diabetes? nurse would expect the patient to report

Glucose tolerance test infertility


HbA dry, irritated skin
Fasting serum glucose undesirable changes in appearance
Fasting glucagon test an increase in height of 2 to 3 inches per year
18. Which of the following is not an indicator of diabetic 3) The nurse is caring for a client with acromegaly. Following
ketoacidosis? a transphenoidal hypophysectomy, the nurse should:

Hyperthermia Monitor the client’s blood sugar


Nausea/Vomiting Suction the mouth and pharynx every hour
Slow and shallow breathing Place the client in low Trendelenburg position
Psychosis leading to dementia Encourage the client to cough
19. Which of the following is not related to a chronic diabetes 4) Of the following disorders, which results from excessive
mellitus condition? secretion of somatotropin?

Atherosclerosis Acromegaly
Neuropathy Cretinism
Glaucoma Dwarfism
Hypotension Adrenogenital syndrome
20. Which of the following conditions is not linked to diabetic 5) Jemma, who weighs 210 lb (95 kg) and has been diagnosed
ketoacidosis? with hyperglycemia tells the nurse that her husband sleeps in
another room because her snoring keeps him awake. The
Cerebral edema nurse notices that she has large hands and a hoarse voice.
Arrhythmias Which of the following would the nurse suspect as a possible
Peptic ulcers cause of the client’s hyperglycemia?
Mucormycosis
Acromegaly
Answers Type 1 diabetes mellitus
1. A. Glucagon Hypothyroidism
2. C. Decreased blood sugar Deficient growth hormone
3. B. FALSE 6) A patient with acromegaly is treated with a transphenoidal
4. B. FALSE hypophysectomy. Postoperatively, the nurse
5. A. TRUE
6. D. Does not require insulin injections ensures that any clear nasal drainage is tested for glucose
7. C. Require insulin 80% of cases maintains the patient flat in bed to prevent cerebrospinal
8. D. Decreased fat metabolism fluid leak
9. B. Poor appetite assists the patient with toothbrushing Q4H to keep the
10. D. Projectile vomiting surgical area clean
11. D. Fever encourages deep breathing and coughing to prevent
12. B. Cyclosporine respiratory complications
13. A. Renin 7) The nurse is planning room assignments for the day. Which
14. B. Alpha cells client should be assigned to a private room if only one is
15. A. Beta cells available?
16. B. Weight gain
17. D. Fasting glucagon test The client with Cushing’s disease
18. C. Slow and shallow breathing The client with diabetes
19. D. Hypotension The client with acromegaly
20. C. Peptic ulcers The client with myxedema

Acromegaly 7 items Answers and Rationales


1. D. a serum sometomedin C (insulin-like growth-factor) D has hyperthyroidism or myxedema and poses no risk
of more than 300. A normal response to growth to others or himself.
hormone secretion is stimulation of the liver to
produce somatomedin C which stimulates growth of Cushing’s Syndrome 22 items
bones and soft tissue. The increased levels of 1) Cushing’s Disease and Cushing’s Sydrome have the same
somatomedin C normally inhibit growth hormone, but cause.
in acromegaly the pituitary gland secretes GH despite
elevated somatomedin C levels.) True
2. C. undesirable changes in appearance . The increased False
production of growth hormone in acromegaly causes 2) You assess a patient with Cushing’s disease. For which
an increase in thickness and width of bones and finding will you notify the physician immediately?
enlargement of soft tissues, resulting in marked
changes in facial features, oily and coarse skin, and Purple striae present on abdomen and thighs
speech difficulties. Height is not increased in adults Weight gain of 1 pound since the previous day
with growth hormone excess because the epiphyses of +1 dependent edema in ankles and calves
the bones are closed, and infertility is not a common Crackles bilaterally in lower lobes of lungs
finding because growth hormone is usually the only 3) As the shift begins, you are assigned these patients. Which
pituitary hormone involved in acromegaly. patient should you assess first?
3. A. Monitor the client’s blood sugar. Growth hormone
levels generally fall rapidly after a hypophysectomy, A 38-year-old patient with Graves’ disease and a heart rate of
allowing insulin levels to rise. The result is 94/minute
hypoglycemia. A 63-year-old patient with type 2 diabetes and fingerstick
4. A. Acromegaly . The patient with acromegaly glucose of 137 mg/dL
demonstrates progressive enlargement of peripheral A 58-year-old patient with hypothyroidism and heart rate of
body parts, most commonly the face, head, hands, and 48/minute
feet. Cretinism occurs as a result of congenital A 49-year-old patient with Cushing’s disease and +1
hypothyroidism.Dwarfism is caused by insufficient dependent edema
secretion of growth hormone during childhood. 4) Which is an obvious sign or sympton of Cushing’s
Adrenogenital syndrome is the result of abnormal Syndrome?
secretion of adrenocortical hormones, especially
androgen. Getting really, really fat.
5. A. Acromegaly . Acromegaly, which is caused by a Losing a lot of weight
pituitary tumor that releases excessive growth Becoming constipated
hormone, is associated with hyperglycemia, Kidney Failure
hypertension, diaphoresis, peripheral neuropathy, and 5) Signs and/or symptoms of Cushing’s Disease include:
joint pain. Enlarged hands and feet are related to (check correct boxes)
lateral bone growth, which is seen in adults with this
disorder. The accompanying soft tissue swelling causes moon face
hoarseness and often sleep apnea. Type 1 diabetes is fatty limbs
usually seen in children, and newly diagnosed persons osteoporosis
are usually very ill and thin. Hypothyroidism isn’t acne
associated with hyperglycemia, nor is growth hormone euphoria
deficiency. 6) The nurse is assessing a postcraniotomy client and finds
6. A. ensures that any clear nasal drainage is tested for the urine output from a catheter is 1500 ml for the 1st hour
glucose. A transphenoidal hypophysectomy involves and the same for the 2nd hour. The nurse should suspect:
entry into the sella turcica through an incision in the
upper lip and gingiva into the floor of the nose and the Cushing’s syndrome
sphenoid sinuses. Postoperative clear nasal drainage Diabetes mellitus
with glucose content indicates CSF leakage from an Adrenal crisis
open connection to the brain, putting the patient at Diabetes insipidus
risk for meningitis. After surgery, the patient is 7) Nurse Ronn is assessing a client with possible Cushing’s
positioned with the head elevated to avoid pressure syndrome. In a client with Cushing’s syndrome, the nurse
on the sella turcica, coughing and straining are avoided would expect to find:
to prevent increased ICP and CSF leakage, and
although mouth care is required Q4H toothbrushing Hypotension.
should not be performed for 7-10post sx. Thick, coarse skin.
7. A. The client with Cushing’s disease. The client with Deposits of adipose tissue in the trunk and dorsocervical
Cushing’s disease has adrenocortical hypersecretion. area.
This increase in the level of cortisone causes the client Weight gain in arms and legs.
to be immune suppressed. In answer B, the client with 8) The pituitary gland is also known as the ____________.
diabetes poses no risk to other clients. The client in
answer C has an increase in growth hormone and Hypophysis
poses no risk to himself or others. The client in answer Infundibulum
Hypothalamus End stage renal disease
Neurohypophysis Cushing’s Disease
9) Which of the following disorders is characterized by a Taking Furosemide and Phenytoin.
group of symptoms produced by an excess of free circulating 17) Which of the following is NOT another name for Cushing’s
cortisol from the adrenal cortex? Syndrome?

Cushing’s syndrome Hypercorticism


Addison’s disease Hyperadrenocorticism
Graves’ disease Itsenko-Cushing syndrome
Hashimoto’s disease Fatty Disease
10) Which gland does Cushing’s Syndrome originate in? 18) Corticosteroids are potent suppressors of the body’s
inflammatory response. Which of the following conditions or
Pituitary actions do they suppress?
Pineal
Pancreas Cushing syndrome.
Gonads Pain receptors.
11) The most common cause of Cushing’s Disease is Immune response.
____________. Neural transmission.
19) Nurse Sugar is assessing a client with Cushing’s syndrome.
Alcohol abuse Which observation should the nurse report to the physician
Steroid use immediately?
Genetic disposition
Adenoma (benign tumor) Pitting edema of the legs
12) A female client with Cushing’s syndrome is admitted to An irregular apical pulse
the medical-surgical unit. During the admission assessment, Dry mucous membranes
nurse Tyzz notes that the client is agitated and irritable, has Frequent urination
poor memory, reports loss of appetite, and appears 20) The nurse is planning room assignments for the day.
disheveled. These findings are consistent with which Which client should be assigned to a private room if only one
problem? is available?

Depression The client with Cushing’s disease


Neuropathy The client with diabetes
Hypoglycemia The client with acromegaly
Hyperthyroidism The client with myxedema
13) Cushing’s Syndrome is a hormone disorder caused by 21) Which would a person with Cushing’s Syndrome prefer to
high levels of what? eat?

Y Chromosomes Fat Free Yogurt


Fiber in the digestive tract Salad
Cortisol in the blood Bacon Grease
Unprotected sexual contact Fiber Bars
14) In a 29-year-old female client who is being successfully 22) Adrenocorticotropic hormone (ACTH) from the pituitary
treated for Cushing’s syndrome, nurse Lyzette would expect a stimulates the __________ glands to release _________.
decline in:
pineal, cortisol
Serum glucose level. adrenal, cortisol
Hair loss. pineal, TSH
Bone mineralization. adrenal, TSH
Menstrual flow.
15) Nurse Ruth is assessing a client after a thyroidectomy. Answers and Rationales
The assessment reveals muscle twitching and tingling, along 1. B. False . They have the same effect, Disease is a
with numbness in the fingers, toes, and mouth area. The pituitary cause, Syndrome is an adrenal cause
nurse should suspect which complication? 2. D. Crackles bilaterally in lower lobes of lungs . The
presence of crackles in the patient’s lungs indicate
Tetany excess fluid volume doe to excess water and sodium
Hemorrhage reabsorption and may be a symptom of pulmonary
Thyroid storm edema, which must be treated rapidly. Striae (stretch
Laryngeal nerve damage marks), weight gain, and dependent edema are
16) The old woman told John that she has osteoporosis; common findings in patients Cushing’s disease. These
Arthur knew that all of the following factors would contribute findings should be monitored, but are not urgent.
to osteoporosis except Focus: Prioritization
3. C. A 58-year-old patient with hypothyroidism and
Hypothyroidism heart rate of 48/minute . Although patients with
hypothyroidism often have cardiac problems that client experiences a return of menstrual flow, not a
include bradycardia, a heart rate of 48/minute may decline in it.
have significant implications for cardiac output and 15. A. Tetany . Tetany may result if the parathyroid glands
hemodynamic stability. Patients with Graves’ disease are excised or damaged during thyroid surgery.
usually have a rapid heart rate, but 94/minute is Hemorrhage is a potential complication after thyroid
within limits. The diabetic patient may need sliding surgery but is characterized by tachycardia,
scale insulin. This is important but not urgent. Patients hypotension, frequent swallowing, feelings of fullness
with Cushing’s disease frequently have dependent at the incision site, choking, and bleeding. Thyroid
edema. Focus: Prioritization storm is another term for severe hyperthyroidism —
4. A. Getting really, really fat. not a complication of thyroidectomy. Laryngeal nerve
5. A. moon face , C. osteoporosis , D. acne . s/s include damage may occur postoperatively, but its signs
fatty abdomen and thinning limbs, euphoria is not a include a hoarse voice and, possibly, acute airway
s/s obstruction.
6. D. Diabetes insipidus . Diabetes insipidus is an abrupt 16. A. Hypothyroidism
onset of extreme polyuria that commonly occurs in 17. D. Fatty Disease
clients after brain surgery. Cushing’s syndrome is 18. C. Immune response. Corticosteroids suppress
excessive glucocorticoid secretion resulting in sodium eosinophils, lymphocytes, and natural-killer cells,
and water retention. Diabetes mellitus is a inhibiting the natural inflammatory process in an
hyperglycemic state marked by polyuria, polydipsia, infected or injured part of the body. This helps resolve
and polyphagia. Adrenal crisis is undersecretion of inflammation, stabilizes lysosomal membranes,
glucocorticoids resulting in profound hypoglycemia, decreases capillary permeability, and depresses
hypovolemia, and hypotension. phagocytosis of tissues by white blood cells, thus
7. C. Deposits of adipose tissue in the trunk and blocking the release of more inflammatory materials.
dorsocervical area. Because of changes in fat Excessive corticosteroid therapy can lead to Cushing
distribution, adipose tissue accumulates in the trunk, syndrome.
face (moonface), and dorsocervical areas (buffalo 19. B. An irregular apical pulse . Because Cushing’s
hump). Hypertension is caused by fluid retention. Skin syndrome causes aldosterone overproduction, which
becomes thin and bruises easily because of a loss of increases urinary potassium loss, the disorder may
collagen. Muscle wasting causes muscle atrophy and lead to hypokalemia. Therefore, the nurse should
thin extremities. immediately report signs and symptoms of
8. A. Hypophysis hypokalemia, such as an irregular apical pulse, to the
9. A. Cushing’s syndrome . The patient with Cushing’s physician. Edema is an expected finding because
syndrome demonstrates truncal obesity, moon face, aldosterone overproduction causes sodium and fluid
acne, abdominal striae, and hypertension. In Addison’s retention. Dry mucous membranes and frequent
disease, the patient experiences chronic adrenocortical urination signal dehydration, which isn’t associated
insufficiency. In Graves’ disease, the patient with Cushing’s syndrome.
experiences hyperthyroidism. The individual with 20. A. The client with Cushing’s disease . The client with
Hashimoto’s disease demonstrates inflammation of Cushing’s disease has adrenocortical hypersecretion.
the thyroid gland, resulting in hypothyroidism. This increase in the level of cortisone causes the client
10. A. Pituitary to be immune suppressed. In answer B, the client with
11. D. Adenoma (benign tumor) diabetes poses no risk to other clients. The client in
12. A. Depression . Agitation, irritability, poor memory, answer C has an increase in growth hormone and
loss of appetite, and neglect of one’s appearance may poses no risk to himself or others. The client in answer
signal depression, which is common in clients with D has hyperthyroidism or myxedema and poses no risk
Cushing’s syndrome. Neuropathy affects clients with to others or himself.
diabetes mellitus — not Cushing’s syndrome. Although 21. C. Bacon Grease
hypoglycemia can cause irritability, it also produces 22. B. adrenal, cortisol
increased appetite, rather than loss of appetite.
Hyperthyroidism typically causes such signs as goiter, Diabetes Insipidus 20 items
nervousness, heat intolerance, and weight loss despite 1) Cyrill with severe head trauma sustained in a car accident
increased appetite. is admitted to the intensive care unit. Thirty-six hours later,
13. C. Cortisol in the blood the client’s urine output suddenly rises above 200 ml/hour,
14. A. Serum glucose level. Hyperglycemia, which develops leading the nurse to suspect diabetes insipidus. Which
from glucocorticoid excess, is a manifestation of laboratory findings support the nurse’s suspicion of diabetes
Cushing’s syndrome. With successful treatment of the insipidus?
disorder, serum glucose levels decline. Hirsutism is
common in Cushing’s syndrome; therefore, with Above-normal urine and serum osmolality levels
successful treatment, abnormal hair growth also Below-normal urine and serum osmolality levels
declines. Osteoporosis occurs in Cushing’s syndrome; Above-normal urine osmolality level, below-normal serum
therefore, with successful treatment, bone osmolality level
mineralization increases. Amenorrhea develops in Below-normal urine osmolality level, above-normal serum
Cushing’s syndrome. With successful treatment, the osmolality level
2) The drug of choice for central diabetes insipidus is Polyuria and polydipsia
desmopressin (DDAVP). What isthis drug’s mechanism of Oliguria and hypoglycemia
action? Weight gain and malaise
10) What electrolyte abnormalities can cause diabetes
Mimics vasopressin and increases kidney water reabsorption insipidus?
Blocks vasopressin and increases kidney water reabsorption
Mimics vasopressin and increases kidney salt excretion Hypercalcemia and hyperkalemia
Blocks vasopressin and increases kidney salt excretion Hypercalcemia and hypokalemia
3) The nurse is assessing a postcraniotomy client and finds Hypocalcemia and hyperkalemia
the urine output from a catheter is 1500 ml for the 1st hour Hypocalcemia and hypokalemia
and the same for the 2nd hour. The nurse should suspect: 11) Nurse Louie is developing a teaching plan for a male
client diagnosed with diabetes insipidus. The nurse should
Cushing’s syndrome include information about which hormone lacking in clients
Diabetes mellitus with diabetes insipidus?
Adrenal crisis
Diabetes insipidus antidiuretic hormone (ADH).
4) A 67-year-old male client has been complaining of sleeping thyroid-stimulating hormone (TSH).
more, increased urination, anorexia, weakness, irritability, follicle-stimulating hormone (FSH).
depression, and bone pain that interferes with her going luteinizing hormone (LH).
outdoors. Based on these assessment findings, nurse Richard 12) A male client with primary diabetes insipidus is ready for
would suspect which of the following disorders? discharge on desmopressin (DDAVP). Which instruction
should nurse Lina provide?
Diabetes mellitus
Diabetes insipidus “Administer desmopressin while the suspension is cold.”
Hypoparathyroidism “Your condition isn’t chronic, so you won’t need to wear a
Hyperparathyroidism medical identification bracelet.”
5) What drugs antagonize the effects of ADH on the renal “You may not be able to use desmopressin nasally if you have
tubules, and thus could causenephrogenic diabetes insipidus? nasal discharge or blockage.”
“You won’t need to monitor your fluid intake and output
Bromocryptine and cabergoline after you start taking desmopressin.”
Hydrochlorothiazide and furosemide 13) You are preparing a 24-year-old patient with diabetes
Cimetidine and verapamil insipidus (DI) for discharge from the hospital. Which
Lithium and demeclocycline statement indicates that the patient needs additional
Acetaminophen and isoniazid teaching?
6) Damage to what organ would cause central diabetes
insipidus? “I will drink fluids equal to the amount of my urine output.”
“I will weigh myself every day using the same scale.”
Kidneys “I will wear my medical alert bracelet at all times.”
Hypothalamus “I will gradually wean myself off the vasopressin.”
Thyroid 14) Which of the following is most suggestive of psychogenic
Parathyroid polydipsia, not diabetesinsipidus?
Pituitary
7) To confirm central diabetes insipidus, post-injection Nocturia
(desmopressin) urine osmolarityshould be what percentage Constant symptoms
of pre-injection osmolarity? 24-hour urine output > 18L
Plasma osmolarity > 295mOsm/kg
< 50% Plasma osmolarity < 280mOsm/kg after a water deprivation
< 75% test
100% (equal) 15) A client is suspected of developing diabetes insipidus.
> 125% Which of the following is the most effective assessment?
> 150%
8) A priority nursing diagnostic for a client admitted to the Taking vital signs every 4 hours
hospital with a diagnosis of diabetes insipidus is: Monitoring blood glucose
Assessing ABG values every other day
Sleep pattern deprivation related nocturia Measuring urine output hourly
Activity intolerance r/t muscle weakness 16) A client with diabetes insipidus is taking Desmopressin
Fluid volume excess r/t intake greater that output acetate (DDAVP). To determine if the drug is effective, the
Risk for impaired skin integrity r/t generalized edema nurse should monitor the client’s:
9) What are the typical presenting signs of diabetes
insipidus? Arterial blood pH
Pulse rate
Hyperglycemia and polyuria Serum glucose
Periorbital ecchymosis and blurred vision Intake and output
17) Which outcome indicates that treatment of a male client 6. E. Pituitary
with diabetes insipidus has been effective? 7. E. > 150%
8. B. Activity intolerance r/t muscle weakness
Fluid intake is less than 2,500 ml/day. 9. A. Hyperglycemia and polyuria
Urine output measures more than 200 ml/hour. 10. B. Hypercalcemia and hypokalemia
Blood pressure is 90/50 mm Hg. 11. A. antidiuretic hormone (ADH). ADH is the hormone
The heart rate is 126 beats/minute. clients with diabetes insipidus lack. The client’s TSH,
18) Adequate fluid replacement and vasopressin replacement FSH, and LH levels won’t be affected.
are objectives of therapy for which of the following disease 12. C. “You may not be able to use desmopressin nasally if
processes? you have nasal discharge or blockage.” Desmopressin
may not be absorbed if the intranasal route is
Diabetes mellitus. compromised. Although diabetes insipidus is treatable,
Diabetes insipidus. the client should wear medical identification and carry
Diabetic ketoacidosis. medication at all times to alert medical personnel in an
Syndrome of inappropriate antidiuretic hormone secretion emergency and ensure proper treatment. The client
(SIADH). must continue to monitor fluid intake and output and
19) The client with a history of diabetes insipidus is admitted receive adequate fluid replacement.
with polyuria, polydipsia, and mental confusion. The priority 13. D. “I will gradually wean myself off the vasopressin.”
intervention for this client is: The patient with permanent DI requires life-long
vasopressin therapy. All of the other statements are
Measure the urinary output appropriate to the home care of this patient. Focus:
Check the vital signs Prioritization
Encourage increased fluid intake 14. C. 24-hour urine output > 18L
Weigh the client 15. D. Measuring urine output hourly. Measuring the urine
20) When caring for a male client with diabetes insipidus, output to detect excess amount and checking the
nurse Juliet expects to administer: specific gravity of urine samples to determine urine
concentration are appropriate measures to determine
vasopressin (Pitressin Synthetic). the onset of diabetes insipidus.
furosemide (Lasix). 16. D. Intake and output . DDAVP replaces the ADH,
regular insulin. facilitating reabsorption of water and consequent
10% dextrose. return of normal urine output and thirst.
17. A. Fluid intake is less than 2,500 ml/day. Diabetes
Answers and Rationales insipidus is characterized by polyuria (up to 8 L/day),
1. D. Below-normal urine osmolality level, above-normal constant thirst, and an unusually high oral intake of
serum osmolality level. In diabetes insipidus, excessive fluids. Treatment with the appropriate drug should
polyuria causes dilute urine, resulting in a below- decrease both oral fluid intake and urine output. A
normal urine osmolality level. At the same time, urine output of 200 ml/hour indicates continuing
polyuria depletes the body of water, causing polyuria. A blood pressure of 90/50 mm Hg and a heart
dehydration that leads to an above-normal serum rate of 126 beats/minute indicate compensation for
osmolality level. For the same reasons, diabetes the continued fluid deficit, suggesting that treatment
insipidus doesn’t cause above-normal urine osmolality hasn’t been effective.
or below-normal serum osmolality levels. 18. B. Diabetes insipidus. Maintaining adequate fluid and
2. A. Mimics vasopressin and increases kidney water replacing vasopressin are the main objectives in
reabsorption treating diabetes insipidus. An excess of antidiuretic
3. D. Diabetes insipidus. Diabetes insipidus is an abrupt hormone leads to SIADH, causing the patient to retain
onset of extreme polyuria that commonly occurs in fluid. Diabetic ketoacidosis is a result of severe insulin
clients after brain surgery. Cushing’s syndrome is insufficiency.
excessive glucocorticoid secretion resulting in sodium 19. B. Check the vital signs . The large amount of fluid loss
and water retention. Diabetes mellitus is a can cause fluid and electrolyte imbalance that should
hyperglycemic state marked by polyuria, polydipsia, be corrected. The loss of electrolytes would be
and polyphagia. Adrenal crisis is undersecretion of reflected in the vital signs. Measuring the urinary
glucocorticoids resulting in profound hypoglycemia, output is important, but the stem already says that the
hypovolemia, and hypotension. client has polyuria. Encouraging fluid intake will not
4. D. Hyperparathyroidism. Hyperparathyroidism is most correct the problem, .Weighing the client is not
common in older women and is characterized by bone necessary at this time.
pain and weakness from excess parathyroid hormone 20. A. vasopressin (Pitressin Synthetic). Because diabetes
(PTH). Clients also exhibit hypercaliuria-causing insipidus results from decreased antidiuretic hormone
polyuria. While clients with diabetes mellitus and (vasopressin) production, the nurse should expect to
diabetes insipidus also have polyuria, they don’t have administer synthetic vasopressin for hormone
bone pain and increased sleeping. Hypoparathyroidism replacement therapy. Furosemide, a diuretic, is
is characterized by urinary frequency rather than contraindicated because a client with diabetes
polyuria. insipidus experiences polyuria. Insulin and dextrose
5. D. Lithium and demeclocycline
are used to treat diabetes mellitus and its Glomuerular injury
complications, not diabetes insipidus. Bleeding of retinal caplillaries
Numbness of feet
Diabetes Mellitus 70 items Impotence
1) Nurse Perry is caring for a female client with type 1 9) What is the number one complication of diabetes?
diabetes mellitus who exhibits confusion, light-headedness,
and aberrant behavior. The client is still conscious. The nurse Diabetic ketoacidosis
should first administer: Obesity
Hypertension
I.M. or subcutaneous glucagon. Cardiovascular disease
I.V. bolus of dextrose 50%. 10) Nurse Noemi administers glucagon to her diabetic client,
15 to 20 g of a fast-acting carbohydrate such as orange juice. then monitors the client for adverse drug reactions and
10 U of fast-acting insulin. interactions. Which type of drug interacts adversely with
2) A client with diabetes mellitus has a prescription for glucagon?
Glucotrol XL (glipizide). The client should be instructed to
take the medication: Oral anticoagulants
Anabolic steroids
At bedtime Beta-adrenergic blockers
With breakfast Thiazide diuretics
Before lunch 11) The newly diagnosed diabetic patient asks the nurse why
After dinner he needs to check his feet every day. The nurse’s best
3) The glycosylated hemoglobin of a 40-year-old client with response is….
diabetes mellitus is 2.5%. The nurse understands that:
To prevent leg amputation.
The client can have a higher-calorie diet. To check for any cuts, sores, or dry cracked skin so they can
The client has good control of her diabetes. be treated early to prevent infection or gangrene.
The client requires adjustment in her insulin dose. To see if they hurt.
The client has poor control of her diabetes. You just need to do it.
4) A patient with severe hypoglycemia arrives at the ED 12) What type of cells secrete glucagon?
unconscious by ambulance. The nurse would first…
Beta cells
Give regular insulin by IV Alpha cells
Give NPH by IV Plasma cells
Give 10-15 g CHO or Orange juice Acinar cells
Give 1 mg glucagon 13) Nurse John is assigned to care for a postoperative male
5) A male client with type 1 diabetes mellitus asks the nurse client who has diabetes mellitus. During the assessment
about taking an oral antidiabetic agent. Nurse Jack explains interview, the client reports that he’s impotent and says he’s
that these medications are only effective if the client: concerned about its effect on his marriage. In planning this
client’s care, the most appropriate intervention would be to:
prefers to take insulin orally.
has type 2 diabetes. Encourage the client to ask questions about personal
has type 1 diabetes. sexuality.
is pregnant and has type 2 diabetes. Provide time for privacy.
6) Which of the following conditions is not linked to diabetic Provide support for the spouse or significant other.
ketoacidosis? Suggest referral to a sex counselor or other appropriate
professional.
Cerebral edema 14) Which are potential complications of diabetes? (Choose
Arrhythmias all that applies)
Peptic ulcers
Mucormycosis Amputations (BKA)
7) Which of the following clinical characteristics is associated Cardiovascular disease
with Type 1 diabetes (previously referred to as insulin- Edema
dependent diabetes mellitus [IDDM])? Peripheral neuropathy
Hyperthyroidism
Presence of islet cell antibodies Retinopathy
Obesity Cardio neuropathy
Rare ketosis Coma
Requirement for oral hypoglycemic agents Nephropathy
8) What are the micro vascular complications of uncontrolled Arteriosclerosis
diabetes? Hypertension
Obesity
Delayed gastric emptying Infections
Diarrhea 15) Which of the following is not an effect of diabetes?
22) A male client with type 1 diabetes mellitus has a highly
Small vessel occlusion elevated glycosylated hemoglobin (Hb) test result. In
Necrosis of extremities discussing the result with the client, nurse Sharmaine would
Ketone Body production be most accurate in stating:
Decreased fat metabolism
16) Which of the following is not an indicator of diabetic “The test needs to be repeated following a 12-hour fast.”
ketoacidosis? “It looks like you aren’t following the prescribed diabetic
diet.”
Hyperthermia “It tells us about your sugar control for the last 3 months.”
Nausea/Vomiting “Your insulin regimen needs to be altered significantly.”
Slow and shallow breathing 23) A male client has just been diagnosed with type 1
Psychosis leading to dementia diabetes mellitus. When teaching the client and family how
17) Dr. Kennedy prescribes glipizide (Glucotrol), an oral diet and exercise affect insulin requirements, Nurse Joy
antidiabetic agent, for a male client with type 2 diabetes should include which guideline?
mellitus who has been having trouble controlling the blood
glucose level through diet and exercise. Which medication “You’ll need more insulin when you exercise or increase your
instruction should the nurse provide? food intake.”
“You’ll need less insulin when you exercise or reduce your
“Be sure to take glipizide 30 minutes before meals.” food intake.”
“Glipizide may cause a low serum sodium level, so make sure “You’ll need less insulin when you increase your food intake.”
you have your sodium level checked monthly.” “You’ll need more insulin when you exercise or decrease your
“You won’t need to check your blood glucose level after you food intake.”
start taking glipizide.” 24) An agitated, confused female client arrives in the
“Take glipizide after a meal to prevent heartburn.” emergency department. Her history includes type 1 diabetes
18) What type of cells secrete insulin? mellitus, hypertension, and angina pectoris. Assessment
reveals pallor, diaphoresis, headache, and intense hunger. A
Beta cells stat blood glucose sample measures 42 mg/dl, and the client
Alpha cells is treated for an acute hypoglycemic reaction. After recovery,
Plasma cells nurse Lily teaches the client to treat hypoglycemia by
Acinar cells ingesting:
19) A 65-year-old female who has diabetes mellitus and has
sustained a large laceration on her left wrist asks the nurse, 2 to 5 g of a simple carbohydrate.
“How long will it take for my scars to disappear?” which 10 to 15 g of a simple carbohydrate.
statement would be the nurse’s best response? 18 to 20 g of a simple carbohydrate.
25 to 30 g of a simple carbohydrate.
“The contraction phase of wound healing can take 2 to 3 25) Acarbose (Precose), an alpha-glucosidase inhibitor, is
years.” prescribed for a female client with type 2 diabetes mellitus.
“Wound healing is very individual but within 4 months the During discharge planning, nurse Pauleen would be aware of
scar should fade.” the client’s need for additional teaching when the client
“With your history and the type of location of the injury, it’s states:
hard to say.”
“If you don’t develop an infection, the wound should heal any “If I have hypoglycemia, I should eat some sugar, not
time between 1 and 3 years from now.” dextrose.”
20) The nurse enters a patient’s room and sees the patient “The drug makes my pancreas release more insulin.”
breathing rapidly with a fruity breath smell. This is known “I should never take insulin while I’m taking this drug.”
as… “It’s best if I take the drug with the first bite of a meal.”
26) Which of the following is not true about Type I DM?
Trousseau’s
Cullen’s May be linked to autoimmunity
Kussmaul’s Onset usually prior to age 20
Bitot’s Beta islet cells destroyed
21) The nurse is performing wound care on a foot ulcer in a Does not require insulin injections
client with type 1 diabetes mellitus. Which technique 27) Which of the following is caused by insulin release?
demonstrates surgical asepsis?
Increased breakdown of fats
Putting on sterile gloves then opening a container of sterile Increase breakdown of proteins
saline. Decreased blood sugar
Cleaning the wound with a circular motion, moving from Causes glucose to be phosphorylated in kidney
outer circles toward the center. 28) The physician has prescribed NPH insulin for a client with
Changing the sterile field after sterile water is spilled on it. diabetes mellitus. Which statement indicates that the client
Placing a sterile dressing ½” (1.3 cm) from the edge of the knows when the peak action of the insulin occurs?
sterile field.
“I will make sure I eat breakfast within 2 hours of taking my
insulin.” appendectomy
“I will need to carry candy or some form of sugar with me all pernicious anemia
the time.” diabetes mellitus
“I will eat a snack around three o’clock each afternoon.” valve replacement
“I can save my dessert from supper for a bedtime snack.” 37) Of the following types of insulin, which is the most rapid
29) Which of the following is not an accurate test for acting?
diabetes?
Humalog
Glucose tolerance test Regular
HbA NPH
Fasting glucagon test Ultralente
30) A home health nurse is at the home of a client with 38) The nurse’s first action upon finding a patient with mild
diabetes and arthritis. The client has difficulty drawing up hypoglycemia is to…
insulin. It would be most appropriate for the nurse to refer
the client to Call the rapid response team
Give 1 mg of glucagon
A social worker from the local hospital Give 10-15 g of CHO or Orange juice
An occupational therapist from the community center Give insulin
A physical therapist from the rehabilitation agency 39) After taking glipizide (Glucotrol) for 9 months, a male
Another client with diabetes mellitus and takes insulin client experiences secondary failure. Which of the following
31) Which of the following is not true about Type II DM? would the nurse expect the physician to do?

Considered adult onset diabetes Initiate insulin therapy.


Cause unknown may be due to genetics Switch the client to a different oral antidiabetic agent.
Require insulin 80% of cases Prescribe an additional oral antidiabetic agent.
May take a drug that sensitize cells or increase insulin release Restrict carbohydrate intake to less than 30% of the total
32) Which of the following is not an indicator of a caloric intake.
hypoglycemic condition? 40) When caring for a female client with a history of
hypoglycemia, nurse Ruby should avoid administering a drug
Fatigue that may potentiate hypoglycemia. Which drug fits this
Poor appetite description?
Tachycardia
Confusion sulfisoxazole (Gantrisin)
33) Glucagon increases blood levels of glucose by causing mexiletine (Mexitil)
liver to breakdown glycogen. prednisone (Orasone)
lithium carbonate (Lithobid)
TRUE 41) A male client with diabetes mellitus is receiving insulin.
FALSE Which statement correctly describes an insulin unit?
34) A patient with a history of diabetes mellitus is in the
second post-operative day following cholecystectomy. She It’s a common measurement in the metric system.
has complained of nausea and isn’t able to eat solid foods. It’s the basis for solids in the avoirdupois system.
The nurse enters the room to find the patient confused and It’s the smallest measurement in the apothecary system.
shaky. Which of the following is the most likely explanation It’s a measure of effect, not a standard measure of weight or
for the patient’s symptoms? quantity.
42) The physician has prescribed Novalog insulin for a client
Anesthesia reaction. with diabetes mellitus. Which statement indicates that the
Hyperglycemia. client knows when the peak action of the insulin occurs?
Hypoglycemia.
Diabetic ketoacidosis. “I will make sure I eat breakfast within 10 minutes of taking
35) A urine test in an undiagnosed diabetic may show…….. my insulin.”
“I will need to carry candy or some form of sugar with me all
glucose and ketones in the urine the time.”
glucose and high amounts of bilirubin in the urine “I will eat a snack around three o’clock each afternoon.”
ketones in the urine “I can save my dessert from supper for a bedtime snack.”
ketones and adrenaline in the urine 43) Which of the following would not be considered an acute
36) A client has a medical history of rheumatic fever, type 1 effect of diabetes mellitus?
(insulin dependent) diabetes mellitus, hypertension,
pernicious anemia, and appendectomy. She’s admitted to the Polyuria
hospital and undergoes mitral valve replacement surgery. Weight gain
After discharge, the client is scheduled for a tooth extraction. Polydipsia
Which history finding is a major risk factor for infective Polyphagia
endocarditis?
44) Which instruction about insulin administration should The drug is rapidly absorbed and has a fast onset of action
nurse Kate give to a client? 52) Which of the following is not related to a chronic diabetes
mellitus condition?
“Always follow the same order when drawing the different
insulins into the syringe.” Atherosclerosis
“Shake the vials before withdrawing the insulin.” Neuropathy
“Store unopened vials of insulin in the freezer at Glaucoma
temperatures well below freezing.” Hypotension
“Discard the intermediate-acting insulin if it appears cloudy.” 53) During a class on exercise for diabetic clients, a female
45) Which of the following drugs may be given as an client asks the nurse educator how often to exercise. The
immunosuppressant soon after onset of Type I Diabetes? nurse educator advises the clients to exercise how often to
meet the goals of planned exercise?
Torsemide
Cyclosporine At least once a week
Clofibrate At least three times a week
Ceftriaxone At least five times a week
46) As blood glucose decreases glucagon is inhibited. Every day
54) For a diabetic male client with a foot ulcer, the physician
TRUE orders bed rest, a wet-to-dry dressing change every shift, and
FALSE blood glucose monitoring before meals and bedtime. Why
47) Of the following categories of oral antidiabetic agents, are wet-to-dry dressings used for this client?
which exert their primary action by directly stimulating the
pancreas to secrete insulin? They contain exudate and provide a moist wound
environment.
Sulfonylureas They protect the wound from mechanical trauma and
Thiazolidinediones promote healing.
Biguanides They debride the wound and promote healing by secondary
Alpha glucosidase inhibitors intention.
48) The nurse enters a diabetic patient’s room at 11:30 and They prevent the entrance of microorganisms and minimize
notices that the patient is diaphoretic, tachycardic, anxious, wound discomfort.
states she is hungry, and doesn’t remember where she is. 55) A client with type 1 diabetes mellitus has been on a
This patient is most likely showing signs of what? regimen of multiple daily injection therapy. He’s being
converted to continuous subcutaneous insulin therapy. While
hyperglycemic teaching the client bout continuous subcutaneous insulin
hypoglycemic therapy, the nurse would be accurate in telling him the
diabetic ketoacidosis regimen includes the use of:
hyperosmolar hyperglycemic noketotic coma
49) A client with type 1 diabetes mellitus who’s a intermediate and long-acting insulins
multigravida visits the clinic at 27 weeks gestation. The nurse short and long-acting insulins
should instruct the client that for most pregnant women with short-acting only
type 1 diabetes mellitus: short and intermediate-acting insulins
56) Patients with Type 1 diabetes mellitus may require which
Weekly fetal movement counts are made by the mother. of the following changes to their daily routine during periods
Contraction stress testing is performed weekly. of infection?
Induction of labor is begun at 34 weeks’ gestation.
Nonstress testing is performed weekly until 32 weeks’ No changes.
gestation Less insulin.
50) The diabetic patient’s lab work comes back with a pH of More insulin.
7.4, serum blood sugar of 950, serum osmolarity of 460, Oral diabetic agents.
pCO2 of 35, HCO3 of 25. The patient is confused and 57) At a senior citizens meeting a nurse talks with a client
dehydrated. This patient is showing signs and symptoms of…. who has diabetes mellitus Type 1. Which statement by the
client during the conversation is most predictive of a
Diabetic ketoacidosis potential for impaired skin integrity?
hyperosmolar hyperglycemic noketotic coma
Hypoglycemia “I give my insulin to myself in my thighs.”
diabetic neuropathy “Sometimes when I put my shoes on I don’t know where my
51) The nurse teaches the patient about glargine (Lantus), a toes are.”
“peakless” basal insulin including which of the following “Here are my up and down glucose readings that I wrote on
statements? my calendar.”
“If I bathe more than once a week my skin feels too dry.”
Do not mix the drug with other insulins 58) Which of the following is not considered an endocrine
Administer the total daily dosage in two doses. hormone?
Draw up the drug first, then add regular insulin.
Renin the client reports that he’s impotent and says that he’s
Insulin concerned about its effect on his marriage. In planning this
Glucagon client’s care, the most appropriate intervention would be to:
Somatostatin
59) A patient arrives at the ED with a blood sugar of 578, Encourage the client to ask questions about personal
serum osmolarity of 300, pH of 7.3, severe thirst, sexuality
dehydration, and confusion. The patient is breathing rapidly Provide time for privacy
and has a fruity breath smell. This patient has symptoms Provide support for the spouse or significant other
of…… Suggest referral to a sex counselor or other appropriate
professional
Diabetic ketoacidosis 66) Patricia a 20 year old college student with diabetes
hyperosmolar hyperglycemic noketotic coma mellitus requests additional information about the
Hypoglycemia advantages of using a pen like insulin deliverydevices. The
diabetic neuropathy nurse explains that the advantages of these devices over
60) A 27-year-old woman has Type I diabetes mellitus. She syringes includes:
and her husband want to have a child so they consulted her
diabetologist, who gave her information on pregnancy and Accurate dose delivery
diabetes. Of primary importance for the diabetic woman who Shorter injection time
is considering pregnancy should be Lower cost with reusable insulin cartridges
Use of smaller gauge needle.
a review of the dietary modifications that will be necessary. 67) Which of the following clinical characteristics is
early prenatal medical care. associated with Type 2 diabetes (previously referred to as
adoption instead of conception. non-insulin-dependent diabetes mellitus [NIDDM])?
understanding that this is a major health risk to the mother.
61) Clients with diabetes mellitus require frequent vision Can control blood glucose through diet and exercise
assessment. The nurse should instruct the client about which Usually thin at diagnosis
of the following eye problems most likely to be associated Ketosis-prone
with diabetes mellitus? Demonstrate islet cell antibodies
68) The nurse teaches the patient about diabetes including
Cataracts which of the following statements?
Retinopathy
Astigmatism Elevated blood glucose levels contribute to complications of
Glaucoma diabetes, such as diminished vision.
62) Which of the following is not an adverse effect of Sugar is found only in dessert foods.
glucagon? The only diet change needed in the treatment of diabetes is
to stop eating sugar.
Allergic reaction Once insulin injections are started in the treatment of Type 2
Vomiting diabetes, they can never be discontinued.
Nausea 69) Glucagon causes increased blood sugar and causes slow
Fever breakdown of glycogen in the liver.
63) Which of the following is not an adverse effect of oral
hypoglycemics? TRUE
FALSE
Hypoglycemia 70) Insulin inhibits the release of _______.
Headache
Rashes Glucagon
Projectile vomiting ADH
64) A 25-year-old woman is in her fifth month of pregnancy. Beta cells
She has been taking 20 units of NPH insulin for diabetes Somatostatin
mellitus daily for six years. Her diabetes has been well
controlled with this dosage. She has been coming for routine Answers and Rationales
prenatal visits, during which diabetic teaching has been 1. C. 15 to 20 g of a fast-acting carbohydrate such as
implemented. Which of the following statements indicates orange juice. This client is having a hypoglycemic
that the woman understands the teaching regarding her episode. Because the client is conscious, the nurse
insulin needs during her pregnancy? should first administer a fast-acting carbohydrate, such
as orange juice, hard candy, or honey. If the client has
“Are you sure all this insulin won’t hurt my baby?” lost consciousness, the nurse should administer either
“I’ll probably need my daily insulin dose raised.” I.M. or subcutaneous glucagon or an I.V. bolus of
“I will continue to take my regular dose of insulin.” dextrose 50%. The nurse shouldn’t administer insulin
“These finger sticks make my hand sore. Can I do them less to a client who’s hypoglycemic; this action will further
frequently?” compromise the client’s condition.
65) A nurse assigned to care for a postoperative male client 2. B. With breakfast. Glucotrol XL is given once a day with
who has diabetes mellitus. During the assessment interview, breakfast. Answer A is incorrect because the client
would develop hypoglycemia while sleeping. Answers 21. C. Changing the sterile field after sterile water is spilled
C and D are incorrect because the client would develop on it. A sterile field is considered contaminated when it
hypoglycemia later in the day or evening. becomes wet. Moisture can act as a wick, allowing
3. B. The client has good control of her diabetes. The microorganisms to contaminate the field. The outside
client’s diabetes is well under control. Answer A is of containers, such as sterile saline bottles, aren’t
incorrect because it will lead to elevated glycosylated sterile. The containers should be opened before sterile
hemoglobin. Answer C is incorrect because the diet gloves are put on and the solution poured over the
and insulin dose are appropriate for the client. Answer sterile dressings placed in a sterile basin. Wounds
D is incorrect because the desired range for should be cleaned from the most contaminated area to
glycosylated hemoglobin in the adult client is 2.5%– the least contaminated area—for example, from the
5.9% center outward. The outer inch of a sterile field
4. D. Give 1 mg glucagon shouldn’t be considered sterile.
5. B. has type 2 diabetes. Oral antidiabetic agents are 22. C. “It tells us about your sugar control for the last 3
only effective in adult clients with type 2 diabetes. Oral months.” The glycosylated Hb test provides an
antidiabetic agents aren’t effective in type 1 diabetes. objective measure of glycemic control over a 3-month
Pregnant and lactating women aren’t prescribed oral period. The test helps identify trends or practices that
antidiabetic agents because the effect on the fetus is impair glycemic control, and it doesn’t require a fasting
uncertain. period before blood is drawn. The nurse can’t conclude
6. C. Peptic ulcers that the result occurs from poor dietary management
7. A. Presence of islet cell antibodies . Individuals with or inadequate insulin coverage.
Type 1 diabetes often have islet cell antibodies. 23. B. “You’ll need less insulin when you exercise or
Individuals with Type 1 diabetes are usually thin or reduce your food intake.” Exercise, reduced food
demonstrate recent weight loss at the time of intake, hypothyroidism, and certain medications
diagnosis. Individuals with Type 1 diabetes are ketosis- decrease the insulin requirements. Growth, pregnancy,
prone when insulin is absent. Individuals with Type 1 greater food intake, stress, surgery, infection, illness,
diabetes need insulin to preserve life. increased insulin antibodies, and certain medications
8. A. Delayed gastric emptying, C. Glomuerular injury D. increase the insulin requirements.
Bleeding of retinal caplillaries , F. Impotence 24. B. 10 to 15 g of a simple carbohydrate. To reverse
9. D. Cardiovascular disease hypoglycemia, the American Diabetes Association
10. A. Oral anticoagulants . As a normal body protein, recommends ingesting 10 to 15 g of a simple
glucagon only interacts adversely with oral carbohydrate, such as three to five pieces of hard
anticoagulants, increasing the anticoagulant effects. It candy, two to three packets of sugar (4 to 6 tsp), or 4
doesn’t interact adversely with anabolic steroids, beta- oz of fruit juice. If necessary, this treatment can be
adrenergic blockers, or thiazide diuretics. repeated in 15 minutes. Ingesting only 2 to 5 g of a
11. B. To check for any cuts, sores, or dry cracked skin so simple carbohydrate may not raise the blood glucose
they can be treated early to prevent infection or level sufficiently. Ingesting more than 15 g may raise it
gangrene. above normal, causing hyperglycemia.
12. B. Alpha cells 25. A. “If I have hypoglycemia, I should eat some sugar,
13. D. Suggest referral to a sex counselor or other not dextrose.”Acarbose delays glucose absorption, so
appropriate professional. The nurse should refer this the client should take an oral form of dextrose rather
client to a sex counselor or other professional. Making than a product containing table sugar when treating
appropriate referrals is a valid part of planning the hypoglycemia. The alpha-glucosidase inhibitors work
client’s care. The nurse doesn’t normally provide sex by delaying the carbohydrate digestion and glucose
counseling. absorption. It’s safe to be on a regimen that includes
14. A. Amputations (BKA) , B. Cardiovascular disease , D. insulin and an alpha-glucosidase inhibitor. The client
Peripheral neuropathy, F. Retinopathy , G. Cardio should take the drug at the start of a meal, not 30
neuropathy , H. Coma, I. Nephropathy, J. minutes to an hour before.
Arteriosclerosis , M. Infections 26. D. Does not require insulin injections
15. D. Decreased fat metabolism 27. C. Decreased blood sugar
16. C. Slow and shallow breathing 28. C. “I will eat a snack around three o’clock each
17. A. “Be sure to take glipizide 30 minutes before afternoon.” NPH insulin peaks in 8–12 hours, so a
meals.”The client should take glipizide twice a day, 30 snack should be offered at that time. NPH insulin
minutes before a meal, because food decreases its onsets in 90–120 minutes, so answer A is incorrect.
absorption. The drug doesn’t cause hyponatremia and Answer B is untrue because NPH insulin is time
therefore doesn’t necessitate monthly serum sodium released and does not usually cause sudden
measurement. The client must continue to monitor the hypoglycemia.
blood glucose level during glipizide therapy. 29. C. Fasting glucagon test
18. A. Beta cells 30. B. An occupational therapist from the community
19. C. “With your history and the type of location of the center. An occupational therapist can assist a client to
injury, it’s hard to say.”. Wound healing in a client with improve the fine motor skills needed to prepare an
diabetes will be delayed. Providing the client with a insulin injection.
time frame could give the client false information. 31. C. Require insulin 80% of cases
20. C. Kussmaul’s 32. B. Poor appetite
33. A. TRUE molecules may be damaged. Intermediate-acting
34. C. Hypoglycemia. A post-operative diabetic patient insulin is normally cloudy.
who is unable to eat is likely to be suffering from 45. B. Cyclosporine
hypoglycemia. Confusion and shakiness are common 46. B. FALSE
symptoms. An anesthesia reaction would not occur on 47. A. Sulfonylureas . A functioning pancreas is necessary
the second post-operative day. Hyperglycemia and for sulfonylureas to be effective.
ketoacidosis do not cause confusion and shakiness. 48. B. hypoglycemic
35. A. glucose and ketones in the urine 49. D. Nonstress testing is performed weekly until 32
36. D. valve replacement. A heart valve prosthesis, such as weeks’ gestation. For most clients with type 1 diabetes
a mitral valve replacement, is a major risk factor for mellitus, nonstress testing is done weekly until 32
infective endocarditis. Other risk factors include a weeks’ gestation and twice a week to assess fetal well-
history of heart disease (especially mitral valve being.
prolapse), chronic debilitating disease, IV drug abuse, 50. B. hyperosmolar hyperglycemic noketotic coma
and immunosuppression. Although diabetes mellitus 51. A. Do not mix the drug with other insulins . Because
may predispose a person to cardiovascular disease, it glargine is in a suspension with a pH of 4, it cannot be
isn’t a major risk factor for infective endocarditis, nor mixed with other insulins because this would cause
is an appendectomy or pernicious anemia. precipitation. When administering glargine (Lantus)
37. A. Humalog . The onset of action of rapid-acting insulin it is very important to read the label carefully
Humalog is within 10-15 minutes. The onset of action and to avoid mistaking Lantus insulin for Lente insulin
of short-acting regular insulin is 30 minutes-1 hour. and vice versa.
The onset of action of intermediate acting NPH is 3-4 52. D. Hypotension
hours. The onset of action of long-acting Ultralente is 53. B. At least three times a week . Diabetic clients must
6-8 hours. exercise at least three times a week to meet the goals
38. C. Give 10-15 g of CHO or Orange juice of planned exercise — lowering the blood glucose
39. B. Switch the client to a different oral antidiabetic level, reducing or maintaining the proper weight,
agent. Many clients (25% to 60%) with secondary increasing the serum high-density lipoprotein level,
failure respond to a different oral antidiabetic agent. decreasing serum triglyceride levels, reducing blood
Therefore, it wouldn’t be appropriate to initiate insulin pressure, and minimizing stress. Exercising once a
therapy at this time. However, if a new oral week wouldn’t achieve these goals. Exercising more
antidiabetic agent is unsuccessful in keeping glucose than three times a week, although beneficial, would
levels at an acceptable level, insulin may be used in exceed the minimum requirement.
addition to the antidiabetic agent. 54. C. They debride the wound and promote healing by
40. A. sulfisoxazole (Gantrisin). Sulfisoxazole and other secondary intention. For this client, wet-to-dry
sulfonamides are chemically related to oral dressings are most appropriate because they clean the
antidiabetic agents and may precipitate hypoglycemia. foot ulcer by debriding exudate and necrotic tissue,
Mexiletine, an antiarrhythmic, is used to treat thus promoting healing by secondary intention. Moist,
refractory ventricular arrhythmias; it doesn’t cause transparent dressings contain exudate and provide a
hypoglycemia. Prednisone, a corticosteroid, is moist wound environment. Hydrocolloid dressings
associated with hyperglycemia. Lithium may cause prevent the entrance of microorganisms and minimize
transient hyperglycemia, not hypoglycemia. wound discomfort. Dry sterile dressings protect the
41. D. It’s a measure of effect, not a standard measure of wound from mechanical trauma and promote healing.
weight or quantity. An insulin unit is a measure of 55. C. short-acting only. Continuous subcutaneous insulin
effect, not a standard measure of weight or quantity. regimen uses a basal rate and boluses of short-acting
Different drugs measured in units may have no insulin. Multiple daily injection therapy uses a
relationship to one another in quality or quantity. combination of short-acting and intermediate or long-
42. A. “I will make sure I eat breakfast within 10 minutes acting insulins.
of taking my insulin.” Novalog insulin onsets very 56. C. More insulin. During periods of infection or illness,
quickly, so food should be available within 10–15 patients with Type 1 diabetes may need even more
minutes of taking the insulin. Answer B does not insulin to compensate for increased blood glucose
address a particular type of insulin, so it is incorrect. levels.
NPH insulin peaks in 8–12 hours, so a snack should be 57. B. “Sometimes when I put my shoes on I don’t know
eaten at the expected peak time. It may not be 3 p.m. where my toes are.” Peripheral neuropathy can lead to
as stated in answer C. Answer D is incorrect because lack of sensation in the lower extremities. Clients do
there is no need to save the dessert until bedtime. not feel pressure and/or pain and are at high risk for
43. B. Weight gain skin impairment.
44. A. “Always follow the same order when drawing the 58. A. Renin
different insulins into the syringe.” The client should 59. A. Diabetic ketoacidosis
be instructed always to follow the same order when 60. B. early prenatal medical care. A review of dietary
drawing the different insulins into the syringe. Insulin modifications is important once the woman is
should never be shaken because the resulting froth pregnant. However, it is not of primary importance
prevents withdrawal of an accurate dose and may when considering pregnancy. Pregnancy makes
damage the insulin protein molecules. Insulin also metabolic control of diabetes more difficult. It is
should never be frozen because the insulin protein essential that the client start prenatal care early so
that potential complications can be controlled or 2. Clinical manifestations associated with a diagnosis of type
minimized by the efforts of the client and health care 1 DM include all of the following except:
team. The alternative of adoption is not necessary just
because the client is a diabetic. Many diabetic women Hypoglycemia
have pregnancies with successful outcomes if they Hyponatremia
receive good care. While there is some risk to the Ketonuria
pregnant diabetic woman, it is not considered a major Polyphagia
health risk. The greater risk is to the fetus. 3. The lowest fasting plasma glucose level suggestive of a
61. B. Retinopathy diagnosis of DM is:
62. D. Fever
63. D. Projectile vomiting 90mg/dl
64. B. “I’ll probably need my daily insulin dose raised.” The 115mg/dl
client starts to need increased insulin in the second 126mg/dl
trimester. This statement indicates a lack of 180mg/dl
understanding. As a result of placental maturation and 4. Rotation sites for insulin injection should be separated
placental production of lactogen, insulin requirements from one another by 2.5 cm (1 inch) and should be used only
begin increasing in the second trimester and may every:
double or quadruple by the end of pregnancy. The
client starts to need increased insulin in the second Third day
trimester. This statement indicates a lack of Week
understanding. Insulin doses depend on blood glucose 2-3 weeks
levels. Finger sticks for glucose levels must be 2-4 weeks
continued. 5. A clinical feature that distinguishes a hypoglycemic
65. D. Suggest referral to a sex counselor or other reaction from a ketoacidosis reaction is:
appropriate professional. The nurse should refer this
client to a sex counselor or other professional. Making Blurred vision
appropriate referrals is a valid part of planning the Diaphoresing
client’s care. The nurse doesn’t normally provide sex Nausea
counseling. Therefore, providing time for privacy and Weakness
providing support for the spouse or significant other 6. Clinical nursing assessment for a patient with
are important, but not as important as referring the microangiopathy who has manifested impaired peripheral
client to a sex counselor. arterial circulation includes all of the following except:
66. A. Accurate dose delivery. These devices are more
accurate because they are easily to used and have Integumentary inspection for the presence of brown spots on
improved adherence in insulin regimens by young the lower extremities
people because the medication can be administered Observation for paleness of the lower extremities
discreetly. Observation for blanching of the feet after the legs are
67. A. Can control blood glucose through diet and elevated for 60 seconds
exercise . Oral hypoglycemic agents may improve Palpation for increased pulse volume in the arteries of the
blood glucose levels if dietary modification and lower extremities
exercise are unsuccessful. Individuals with Type 2 7. The nurse expects that a type 1 diabetic may receive ____
diabetes are usually obese at diagnosis. Individuals of his or her morning dose of insulin preoperatively:
with Type 2 diabetes rarely demonstrate ketosis,
except in stress or infection. Individuals with Type 2 10-20%
diabetes do not demonstrate islet cell antibodies. 25-40%
68. A. Elevated blood glucose levels contribute to 50-60%
complications of diabetes, such as diminished vision. 85-90%
When blood glucose levels are well controlled, the 8. Albert, a 35-year-old insulin dependent diabetic, is
potential for complications of diabetes is reduced. admitted to the hospital with a diagnosis of pneumonia. He
69. B. FALSE has been febrile since admission. His daily insulin
70. A. Glucagon requirement is 24 units of NPH. Every morning Albert is given
NPH insulin at 0730. Meals are served at 0830, 1230, and
Diabetes Mellitus 2 95 items 1830. The nurse expects that the NPH insulin will reach its
1. Knowing that gluconeogenesis helps to maintain blood maximum effect (peak) between the hours of:
levels, a nurse should:
1130 and 1330
Document weight changes because of fatty acid mobilization 1330 and 1930
Evaluate the patient’s sensitivity to low room temperatures 1530 and 2130
because of decreased adipose tissue insulation 1730 and 2330
Protect the patient from sources of infection because of 9. A bedtime snack is provided for Albert. This is based on the
decreased cellular protein deposits knowledge that intermediate-acting insulins are effective for
Do all of the above an approximate duration of:
6-8 hours 16. A nurse is caring for a client admitted to the ER with DKA.
10-14 hours In the acute phase the priority nursing action is to prepare to:
16-20 hours
24-28 hours Administer regular insulin intravenously
10. Albert refuses his bedtime snack. This should alert the Administer 5% dextrose intravenously
nurse to assess for: Correct the acidosis
Apply an electrocardiogram monitor.
Elevated serum bicarbonate and a decreased blood pH. 17. A nurse performs a physical assessment on a client with
Signs of hypoglycemia earlier than expected. type 2 DM. Findings include a fasting blood glucose of
Symptoms of hyperglycemia during the peak time of NPH 120mg/dl, temperature of 101, pulse of 88, respirations of
insulin. 22, and a bp of 140/84. Which finding would be of most
Sugar in the urine concern of the nurse?
11. A client is taking NPH insulin daily every morning. The
nurse instructs the client that the most likely time for a Pulse
hypoglycemic reaction to occur is: BP
Respiration
2-4 hours after administration Temperature
6-14 hours after administration 18. A client with type 1 DM calls the nurse to report recurrent
16-18 hours after administration episodes of hypoglycemia with exercise. Which statement by
18-24 hours after administration the client indicated an inadequate understanding of the peak
12. An external insulin pump is prescribed for a client with action of NPH insulin and exercise?
DM. The client asks the nurse about the functioning of the
pump. The nurse bases the response on the information that “The best time for me to exercise is every afternoon.”
the pump: “The best time for me to exercise is right after I eat.”
“The best time for me to exercise is after breakfast.”
Gives small continuous dose of regular insulin “The best time for me to exercise is after my morning snack.”
subcutaneously, and the client can self-administer a bolus 19. A client with diabetes mellitus visits a health care clinic.
with an additional dosage from the pump before each meal. The client’s diabetes previously had been well controlled with
Is timed to release programmed doses of regular or NPH glyburide (Diabeta), 5 mg PO daily, but recently the fasting
insulin into the bloodstream at specific intervals. blood glucose has been running 180-200mg/dl. Which
Is surgically attached to the pancreas and infuses regular medication, if added to the clients regimen, may have
insulin into the pancreas, which in turn releases the insulin contributed to the hyperglycemia?
into the bloodstream.
Continuously infuses small amounts of NPH insulin into the Prednisone (Deltasone)
bloodstream while regularly monitoring blood glucose levels. Atenolol (Tenormin)
13. A client with a diagnosis of diabetic ketoacidosis (DKA) is Phenelzine (Nardil)
being treated in the ER. Which finding would a nurse expect Allopurinol (Zyloprim)
to note as confirming this diagnosis? 20. Glucose is an important molecule in a cell because this
molecule is primarily used for:
Elevated blood glucose level and a low plasma bicarbonate
Decreased urine output Extraction of energy
Increased respirations and an increase in pH Synthesis of protein
Comatose state Building of genetic material
14. A client with DM demonstrates acute anxiety when first Formation of cell membranes.
admitted for the treatment of hyperglycemia. The most 21. When a client is first admitted with hyperglycemic
appropriate intervention to decrease the client’s anxiety hyperosmolar nonketotic syndrome (HHNS), the nurse’s
would be to: priority is to provide:

Administer a sedative Oxygen


Make sure the client knows all the correct medical terms to Carbohydrates
understand what is happening. Fluid replacement
Ignore the signs and symptoms of anxiety so that they will Dietary instruction
soon disappear 22. The nurse is admitting a client with hypoglycemia. Identify
Convey empathy, trust, and respect toward the client. the signs and symptoms the nurse should expect. Select all
15. A nurse is preparing a plan of care for a client with DM that apply.
who has hyperglycemia. The priority nursing diagnosis would
be: Thirst
Palpitations
High risk for deficient fluid volume Diaphoresis
Deficient knowledge: disease process and treatment Slurred speech
Imbalanced nutrition: less than body requirements Hyperventilation
Disabled family coping: compromised. 23. When a client is in diabetic ketoacidosis, the insulin that
would be administered is:
Humulin N
Human NPH insulin 31. A client with DM states, “I cannot eat big meals; I prefer
Human regular insulin to snack throughout the day.” The nurse should carefully
Insulin lispro injection explain that the:
Insulin glargine injection
24. The nurse recognizes that additional teaching is necessary Regulated food intake is basic to control
when the client who is learning alternative site testing (AST) Salt and sugar restriction is the main concern
for glucose monitoring says: Small, frequent meals are better for digestion
Large meals can contribute to a weight problem
“I need to rub my forearm vigorously until warm before 32. A client with DM has an above-knee amputation because
testing at this site.” of severe peripheral vascular disease, Two days following
“The fingertip is preferred for glucose monitoring if surgery, when preparing the client for dinner, it is the nurse’s
hyperglycemia is suspected.” primary responsibility to:
“I have to make sure that my current glucose monitor can be
used at an alternate site.” Check the client’s serum glucose level
“Alternate site testing is unsafe if I am experiencing a rapid Assist the client out of bed to the chair
change in glucose levels.” Place the client in a high-fowlers position
25. Which adaptations should the nurse caring for a client Ensure that the client’s residual limb is elevated.
with diabetic ketoacidosis expect the client to exhibit? Select 33. Which of the following nursing interventions should be
all that apply: taken for a client who complains of nausea and vomits one
hour after taking his glyburide (DiaBeta)?
Sweating
Low PCO2 Give glyburide again
Retinopathy Give subcutaneous insulin and monitor blood glucose
Acetone breath Monitor blood glucose closely, and look for signs of
Elevated serum bicarbonate hypoglycemia.
26. A client’s blood gases reflect diabetic acidosis. The nurse Monitor blood glucose, and assess for signs of hyperglycemia.
should expect: 34. Which of the following chronic complications is
associated with diabetes?
Increased pH
Decreased PO2 Dizziness, dyspnea on exertion, and coronary artery disease.
Increased PCO2 Retinopathy, neuropathy, and coronary artery disease
Decreased HCO3 Leg ulcers, cerebral ischemic events, and pulmonary infarcts
27. The nurse knows that glucagon may be given in the Fatigue, nausea, vomiting, muscle weakness, and cardiac
treatment of hypoglycemia because it: arrhythmia’s
35. Rotating injection sites when administering insulin
Inhibits gluconeogenesis prevents which of the following complications?
Stimulates the release of insulin
Increases blood glucose levels Insulin edema
Provides more storage of glucose. Insulin lipodystrophy
28. A client with type 1 DM has a fingerstick glucose level of Insulin resistance
258mg/dl at bedtime. An order for sliding scale insulin exists. Systemic allergic reactions
The nurse should: 36. Which of the following methods of insulin administration
would be used in the initial treatment of hyperglycemia in a
Call the physician client with diabetic ketoacidosis?
Encourage the intake of fluids
Administer the insulin as ordered Subcutaneous
Give the client ½ c. of orange juice Intramuscular
29. The physician orders 36 units of NPH and 12 units of IV bolus only
regular insulin. The nurse plans to administer these drugs in 1 IV bolus, followed by continuous infusion.
syringe. Identify the steps in this procedure by listing them in 37. Insulin forces which of the following electrolytes out of
priority order. the plasma and into the cells?

Inject air equal to NPH dose into NPH vial Calcium


Invert regular insulin bottle and withdraw regular insulin dose Magnesium
Inject air equal to regular dose into regular dose Phosphorus
Invert NPH vial and withdraw NPH dose. Potassium
30. The insulin that has the most rapid onset of action would 38. Which of the following causes of HHNS is most common?
be:
Insulin overdose
Lente Removal of the adrenal gland
Lispro Undiagnosed, untreated hyperpituitarism
Ultralente Undiagnosed, untreated diabetes mellitus
39. A client is in DKA, secondary to infection. As the condition 46. Jansen receives metformin (Glucophage). What will the
progresses, which of the following symptoms might the nurse best plan of the nurse include with regard to patient
see? education with this drug? Select all that apply.

Kussmaul’s respirations and a fruity odor on the breath It stimulates the pancreas to produce more insulin.
Shallow respirations and severe abdominal pain It must be taken with meals.
Decreased respirations and increased urine output. It decreases sugar production in the liver.
Cheyne-stokes respirations and foul-smelling urine It inhibits absorption of carbohydrates.
40. Clients with type 1 diabetes may require which of the It reduces insulin resistance.
following changes to their daily routine during periods of 47. Serafica who has diabetes mellitus type 1 is found
infection? unresponsive in the clinical setting. Which nursing action is a
priority? arrange from 1 to 4.
No changes
Less insulin Treat the client for hypoglycemia.
More insulin Call the physician STAT.
Oral antidiabetic agents Assess the vital signs.
41. Marlisa has been diagnosed with diabetes mellitus type 1. Call a code.
She asks Nurse Errol what this means. What is the best 1, 2, 3, 4
response by the nurse? Select all that apply. 1, 3, 2, 4
3, 1, 2, 4
“Your alpha cells should be able to secrete insulin, but 4, 3, 2, 1
cannot.” 48. Serge who has diabetes mellitus is taking oral agents, and
“The exocrine function of your pancreas is to secrete insulin.” is scheduled for a diagnostic test that requires him to be
“Without insulin, you will develop ketoacidosis (DKA).” NPO. What is the best plan of the nurse with regard to giving
“The endocrine function of your pancreas is to secrete the client his oral medications?
insulin.”
“It means your pancreas cannot secrete insulin.” Administer the oral agents immediately after the test.
42. Dr. Shrunk orders intravenous (IV) insulin for Rita, a client Notify the the diagnostic department and request orders.
with a blood sugar of 563. Nurse AJ administers insulin lispro Notify the physician and request orders.
(Humalog) intravenously (IV). What does the best evaluation Administer the oral agents with a sip of water before the test.
of the nurse reveal? Select all that apply. 49. A client diagnosed with type 1 diabetes receives insulin.
He asks the nurse why he can’t just take pills instead. What is
The nurse could have given the insulin subcutaneously. the best response by the nurse?
The nurse should have contacted the physician.
The nurse should have used regular insulin (Humulin R). “Insulin must be injected because it needs to work quickly.”
The nurse used the correct insulin. “Insulin can’t be in a pill because it is destroyed in stomach
43. Ben injects his insulin as prescribed, but then gets busy acid.”
and forgets to eat. What will the best assessment of the “Have you talked to your doctor about taking pills instead?”
nurse reveal? “I know it is tough, but you will get used to the shots soon.”
50. Nurse Andy has finished teaching a client with diabetes
The client will be very tasty. mellitus how to administer insulin. He evaluates the learning
The client will complain of nausea. has occurred when the client makes which statement?
The client will need to urinate.
The client will have moist skin. “I should check my blood sugar immediately prior to the
44. A clinical instructor teaches a class for the public about administration.”
diabetes mellitus. Which individual does the nurse assess as “I should provide direct pressure over the site following the
being at highest risk for developing diabetes? injection.”
“I should use the abdominal area only for insulin injections.”
The 50-year-old client who does not get any physical exercise “I should only use calibrated insulin syringe for the
The 56-year-old client who drinks three glasses of wine each injections.”
evening 51. Genevieve has diabetes type 1 and receives insulin for
The 42 year-old client who is 50 pounds overweight glycemic control. She tells the nurse that she likes to have a
The 38 year-old client who smokes one pack of cigarettes per glass of wine with dinner. What will the best plan of the
day nurse for client education include?
45. Steven John has type 1 diabetes mellitus and receives
insulin. Which laboratory test will the nurse assess? The alcohol could cause pancreatic disease.
The alcohol could cause serious liver disease.
Potassium The alcohol could predispose you to hypoglycemia.
AST (aspartate aminotransferase) The alcohol could predispose you to hyperglycemia.
Serum amylase 52. Dr. Wijangco orders insulin lispro (Humalog) 10 units for
Sodium Alicia, a client with diabetes mellitus. When will the nurse
administer this medication?
When the client is eating
Thirty minutes before meals Induce hypoglycemia by decreasing insulin sensitivity.
fifteen minutes before meals Improve insulin sensitivity and decrease hyperglycemia.
When the meal trays arrive on the floor Stimulate the beta cells of the pancreas to secrete insulin.
53. Nurse Matt makes a home visit to the client with diabetes Decrease insulin sensitivity by enhancing glucose uptake.
mellitus. During the visit, Nurse Matt notes the client’s 60. Rosemary has been taking Glargine (Lantus) to treat her
additional insulin vials are not refrigerated. What is the best condition. One of the benefits of Glargine (Lantus) insulin is
action by the nurse at this time? its ability to:

Instruct the client to label each vial with the date when Release insulin rapidly throughout the day to help control
opened. basal glucose.
Tell the client there is no need to keep additional vials. Release insulin evenly throughout the day and control basal
Have the client place the insulin vials in the refrigerator. glucose levels.
Have the client discard the vials. Simplify the dosing and better control blood glucose levels
54. During the morning rounds, Nurse AJ accompanied the during the day.
physician in every patient’s room. The physician writes orders Cause hypoglycemia with other manifestation of other
for the client with diabetes mellitus. Which order would the adverse reactions.
nurse validate with the physician? 61. A 50-year-old widower is admitted to the hospital with a
diagnosis of diabetes mellitus and complaints of rapid-onset
Use Humalog insulin for sliding scale coverage. weight loss, elevated blood glucose levels, and polyphagia,
Metformin (Glucophage) 1000 mg per day in divided doses. the gerontology nurse should anticipate which of the
Administer regular insulin 30 minutes prior to meals. following secondary medical diagnoses?
Lantus insulin 20U BID.
55. Gary has diabetes type 2. Nurse Martha has taught him Impaired glucose tolerance
about the illness and evaluates learning has occurred when Gestational diabetes mellitus
the client makes which statement? Pituitary tumor
Pancreatic tumor
“My cells have increased their receptors, but there is enough 62. An older woman with diabetes mellitus visits the clinic
insulin.” concerning her condition. Of which of the following
“My peripheral cells have increased sensitivity to insulin.” symptoms might an older woman with diabetes mellitus
“My beta cells cannot produce enough insulin for my cells.” complain?
“My cells cannot use the insulin my pancreas makes.”
56. The principal goals of therapy for older patients who have Anorexia
poor glycemic control are: Pain intolerance
Weight loss
Enhancing quality of life. Perineal itching
Decreasing the chance of complications. 63. Gregory is a 52-year-old man identified as high-risk for
Improving self-care through education. diabetes mellitus. Which laboratory test should a nurse
All of the above. anticipate a physician would order for him? (Select all that
57. Which of the following is accurate pertaining to physical apply.)
exercise and type 1 diabetes mellitus?
Fasting Plasma Glucose (FPG)
Physical exercise can slow the progression of diabetes Two-hour Oral Glucose Tolerance Test (OGTT)
mellitus. Glycosylated hemoglobin (HbA1C)
Strenuous exercise is beneficial when the blood glucose is Finger stick glucose three times daily
high. 64. According to the National Diabetes Statistics Report,
Patients who take insulin and engage in strenuous physical diabetes remains as one of the leading causes of death in the
exercise might experience hyperglycemia. United States since 2010. Which of the following factors are
Adjusting insulin regimen allows for safe participation in all risks for the development of diabetes mellitus? (Select all
forms of exercise. that apply.)
58. Harry is a diabetic patient who is experiencing a reaction
of alternating periods of nocturnal hypoglycemia and Age over 45 years
hyperglycemia. The patient might be manifesting which of Overweight with a waist/hip ratio >1
the following? Having a consistent HDL level above 40 mg/dl
Maintaining a sedentary lifestyle
Uncontrolled diabetes 65. During a visit in the hospital, the student nurses are asked
Somogyi phenomenon which of the following persons would most likely be
Brittle diabetes diagnosed with diabetes mellitus. They are correct if they
Diabetes insipidus answered a 44-year-old:
59. Dr. Hugo has prescribed sulfonylureas for Rebecca in the
management of diabetes mellitus type 2. As a nurse, you Caucasian woman.
know that the primary purpose of sulfonylureas, such as Asian woman.
long-acting glyburide (Micronase), is to: African-American woman.
Hispanic male. retinopathy
66. An ailing 70-year-old woman with a diagnosis of type 2 atherosclerosis
diabetes mellitus has been ill with pneumonia. The client’s glycosuria
intake has been very poor, and she is admitted to the hospital acidosis
for observation and management as needed. What is the 73. Joko has recently been diagnosed with Type I diabetes
most likely problem with this patient? and asks Nurse Jessica for help formulating a nutrition plan.
Which of the following recommendations would the nurse
Insulin resistance has developed. make to help the client increase calorie consumption to
Diabetic ketoacidosis is occurring. offset absorption problems?
Hypoglycemia unawareness is developing.
Hyperglycemic hyperosmolar non-ketotic coma. Eat small meals with two or three snacks throughout the day
67. Daniel is diagnosed of having hyperthyroidism (Graves’ to keep blood glucose levels steady
disease). Which of the following is a drug of choice for his Increase consumption of simple carbohydrates
condition? Eating small meals with two or three snacks may be more
helpful in maintaining blood glucose levels than three large
Furosemide (Lasix) meals.
Digoxin (Lanoxin) Skip meals to help lose weight
Propranolol (Inderal) 74. Billy is being asked concerning his health in the
Propylthiouracil (PTU) emergency department. When obtaining a health history
68. Which of the following medications are most likely to from a patient with acute pancreatitis, the nurse asks the
cause hypothyroidism? (Select all that apply.) patient specifically about a history of

Acetylsalicylic acid (aspirin) alcohol use.


Furosemide (Lasix) cigarette smoking.
Docusate sodium (Colace) diabetes mellitus.
Rifampin (Rifadin) high-protein diet.
69. After visiting the physician, Angela found out that she has 75. Nurse Shey is educating a pregnant client who has
a thyroid problem. In line with her condition, which of the gestational diabetes. Which of the following statements
following diagnostic studies is done to determine the size and should the nurse make to the client? Select all that apply.
composition of the thyroid gland?
Cakes, candies, cookies, and regular soft drinks should be
Thyroid scan with RAI 123I avoided.
Electrocardiography Gestational diabetes increases the risk that the mother will
Ultrasonography develop diabetes later in life.
Venous duplex Doppler study Gestational diabetes usually resolves after the baby is born.
70. Nurse Gil is caring for a patient with a diagnosis of Insulin injections may be necessary.
hypothyroidism. Which nursing diagnosis should the nurse The baby will likely be born with diabetes
most seriously consider when analyzing the needs of the The mother should strive to gain no more weight during the
patient? pregnancy.
76. The goal for pre-prandial blood glucose for those with
High risk for aspiration related to severe vomiting Type 1 diabetes mellitus is:
Diarrhea related to increased peristalsis
Hypothermia related to slowed metabolic rate <80 mg/dl
Oral mucous membrane, altered related to disease process <130 mg/dl
71. During lecture, the clinical instructor tells the students <180 mg/dl
that 50% to 60% of daily calories should come from <6%
carbohydrates. What should the nurse say about the types of 77. The guidelines for Carbohydrate Counting as medical
carbohydrates that can be eaten? nutrition therapy for diabetes mellitus includes all of the
following EXCEPT:
Try to limit simple sugars to between 10% and 20% of daily
calories. Flexibility in types and amounts of foods consumed
Simple carbohydrates are absorbed more rapidly than Unlimited intake of total fat, saturated fat and cholesterol
complex carbohydrates. Including adequate servings of fruits, vegetables and the
Simple sugars cause rapid spike in glucose levels and should dairy group
be avoided. Applicable to with either Type 1 or Type 2 diabetes mellitus
Simple sugars should never be consumed by someone with 78. The nurse working in the physician’s office is reviewing
diabetes. lab results on the clients seen that day. One of the clients
72. At the time Cherrie Ann found out that the symptoms of who has classic diabetic symptoms had an eight-hour fasting
diabetes were caused by high levels of blood glucose, she plasma glucose (FPG) test done. The nurse realizes that
decided to break the habit of eating carbohydrates. With this, diagnostic criteria developed by the American Diabetes
the nurse would be aware that the client might develop what Association for diabetes include classic diabetic symptoms
complication? plus which of the following fasting plasma glucose levels?
Higher than 106 mg/dl Taking alcohol and/or aspirin with a sulfonylurea drug can
Higher than 126 mg/dl cause development of hypoglycemia
Higher than 140 mg/dl Aspirin and alcohol will cause the stomach to bleed more
Higher than 160 mg/dl when on a sulfonylurea drug
79. When taking a health history, the nurse screens for 84. Which of the following if stated by the nurse is correct
manifestations suggestive of Diabetes Type I. Which of the about Hyperglycemic Hyperosmolar Nonketotic Syndrome
following manifestations are considered the primary (HHNS)?
manifestations of Diabetes Type I and would be most
suggestive and require follow-up investigation? This syndrome occurs mainly in people with Type I Diabetes
It has a higher mortality rate than Diabetic Ketoacidosis
Excessive intake of calories, rapid weight gain, and difficulty The client with HHNS is in a state of overhydration
losing weight This condition develops very rapidly
An increase in three areas: thirst, intake of fluids, and hunger 85. Nurse Robedee is teaching a thin client about the proper
Poor circulation, wound healing, and leg ulcers, methods/techniques when giving insulin. Which one of the
Lack of energy, weight gain, and depression following is proper?
80. The nurse is working with an overweight client who has a
high-stress job and smokes. This client has just received a Pinch the skin up and use a 90 degree angle
diagnosis of Type II Diabetes and has just been started on an Use a 45 degree angle with the skin pinched up
oral hypoglycemic agent. Which of the following goals for the Massage the area of injection after injecting the insulin
client which if met, would be most likely to lead to an Warm the skin with a warmed towel or washcloth prior to
improvement in insulin efficiency to the point the client the injection
would no longer require oral hypoglycemic agents? 86. Nurse Pira is explaining to the client about Type II
Diabetes. Risk factors of such condition include all of the
Comply with medication regimen 100% for 6 months following except:
Quit the use of any tobacco products by the end of three
months Advanced age
Lose a pound a week until weight is in normal range for Physical inactivity
height and exercise 30 minutes daily Obesity
Practice relaxation techniques for at least five minutes five Smoking
times a day for at least five months 87. Blood sugar is well controlled when Hemoglobin A1C is:
81. During a visit in a community, the nurse will recommend
routine screening for diabetes when the person has one or Below 5.7%
more of seven risk criteria. Which of the following persons Between 12%-15%
that the nurse comes in contact with most needs to be Less than 180 mg/dL
screened for diabetes based on the seven risk criteria? Between 90 and 130 mg/dL
88. Which of the following diabetes drugs acts by decreasing
A client with an HDL cholesterol level of 40 mg/dl and a the amount of glucose produced by the liver?
triglyceride level of 300 mg/dl
A woman who is at 90% of standard body weight after Alpha-glucosidase inhibitors
delivering an eight-pound baby Biguanides
A middle-aged Caucasian male Meglitinides
An older client who is hypotensive Sulfonylureas
82. During the admission of a client with diabetic 89. A 39-year-old company driver presents with shakiness,
ketoacidosis, Nurse Kendra will anticipate the physician sweating, anxiety, and palpitations and tells the nurse he has
ordering which of the following types of intravenous solution Type I Diabetes Mellitus. Which of the follow actions should
if the client cannot take fluids orally? the nurse do first?

Lactated Ringer’s solution Inject 1 mg of glucagon subcutaneously.


0.9 normal saline solution Administer 50 mL of 50% glucose I.V.
5% dextrose in water (D5W) Give 4 to 6 oz (118 to 177 mL) of orange juice.
0.45% normal saline solution Give the client four to six glucose tablets.
83. You are doing some teaching with a client who is starting 90. An external insulin pump is prescribed for a client with
on a sulfonylurea antidiabetic agent. The client mentions that diabetes mellitus and the client asks the nurse about the
he usually has a couple of beers each night and takes an functioning of the pump. The nurse bases the response on
aspirin each day to prevent heart attack and/or strokes. the information that the pump:
Which of the following responses would be best on the part
of the nurse? is timed to release programmed doses of regular or NPH
insulin into the bloodstream at specific intervals
As long as you only drink two beers and take one aspirin, this gives a small continuously dose of regular insulin
should not be a problem subcutaneously, and the client can self-administer a bolus
The aspirin is alright but you need to give up drinking any with an additional dose from the pump before each meal
alcoholic beverages continuously infuses small amounts of NPH insulin into the
bloodstream while regularly monitoring blood glucose levels
is surgically attached to the pancreas and infuses regular it’s duration of action is 24 hours. Hypoglycemic
insulin into the pancreas, which in turn releases the insulin reactions most likely occur during peak time.
into the bloodstream 12. Answer: A. An insulin pump provides a small
91. Which of the following persons would most likely be continuous dose of regular insulin subcutaneously
diagnosed with Diabetes Mellitus? A 44-year-old: throughout the day and night, and the client can self-
administer a bolus with additional dosage from the
Caucasian woman. pump before each meal as neede4. Regular insulin is
Asian woman. used in an insulin pump. An external pump is not
African-American woman. attached surgically to the pancreas.
Hispanic male. 13. Answer: A. In diabetic acidiosis the arterial pH is less
92. Which of the following factors are risks for the than 7.35. plasma bicarbonate is less than 15mEq/L,
development of Diabetes Mellitus? Select all that apply. and the blood glucose level is higher than 250mg/dl
and ketones are present in the blood and urine. The
Age over 45 years client would be experiencing polyuria, and Kussmauls
Overweight with a waist/hip ratio >1 respirations would be present. A comatose state may
Having a consistent HDL level above 40 mg/dl occur if DKA is not treated, but coma would not
Maintaining a sedentary lifestyle confirm the diagnosis
93. Anton brought his grandfather to the clinic to confirm his 14. Answer: D. The most appropriate intervention is to
blood sugar levels. Which laboratory test should a nurse address the client’s feelings related to the anxiety.
anticipate a physician would order when an older person is Administering a sedative is not the most appropriate
identified as high-risk for Diabetes Mellitus? Select all that intervention. The nurse should not ignore the client’s
apply. anxious feelings. A client will not relate to medical
terms, particularly when anxiety exists.
Fasting Plasma Glucose (FPG) 15. Answer: A. Increased blood glucose will cause the
Two-hour Oral Glucose Tolerance Test (OGTT) kidneys to excrete the glucose on the urine. This
Glycosylated hemoglobin (HbA1C) glucose is accompanied by fluids and electrolytes,
Finger stick glucose three times daily causing osmotic diuresis leading to dehydration. This
94. A patient received 6 units of regular insulin 3 hours ago. fluid loss must be replaced when it becomes severe.
The nurse would be MOST concerned if which of the Options B, C, and D are not related specifically to the
following was observed? issue of the question.
16. Answer: A. lack (absolute or relative) of insulin is the
kussmaul respirations and diaphoresis primary cause of DK1. Treatment consists of insulin
anorexia and lethargy administration (regular insulin), IV fluid administration
diaphoresis and trembling (normal saline initially), and potassium replacement,
headache and polyuria followed by correcting acidosis. Applying an
95. Mr. Wesley is newly diagnosed with Type I DM and is electrocardiogram monitor is not a priority action.
being seen by the home health nurse. The doctors orders 17. Answer: D. An elevated temperature may indicate
include: 1200 calorie ADA diet, 15 units NPH insulin before infection. Infection is a leading cause of hyperglycemic
breakfast, and check blood sugar qid. When the nurse visits hyperosmolar nonketotic syndrome or diabetic
the patient at 5 pm, the nurse observes the man performing ketoacidosis.
blood sugar analysis. The result is 50 mg/dL. The nurse would 18. Answer: A. A hypoglycemic reaction may occur in the
expect the patient to be response to increased exercise. Clients should avoid
exercise during the peak time of insulin. NPH insulin
confused with cold, clammy skin and pulse of 110 peaks at 6-14 hours; therefore afternoon exercise will
lethargic with hot dry skin and rapid deep respirations occur during the peak of the medication. Options B, C,
alert and cooperative with BP of 130/80 and respirations of and D do not address peak action times.
12 19. Answer: A. Prednisone may decrease the effect of oral
short of breath, with distended neck veins and bounding hypoglycemics, insulin, diuretics, and potassium
pulse of 96. supplements.
20. Answer: A. Glucose catabolism is the main pathway for
Answers and Rationales cellular energy production.
1. Answer: D. 21. Answer: C. As a result of osmotic pressures created by
2. Answer: A. increased serum glucose, the cells become dehydrated;
3. Answer: C. the client must receive fluid and then insulin.
4. Answer: C. 22. Answer: B, C, D. Palpitations, an adrenergic symptom,
5. Answer: B. occur as the glucose levels fall; the sympathetic
6. Answer: D. nervous system is activated and epinephrine and
7. Answer: C. norepinephrine are secreted causing this response.
8. Answer: B. Diaphoresis is a sympathetic nervous system response
9. Answer: C. that occurs as epinephrine and norepinephrine are
10. Answer: B. released. Slurred speech is a neuroglycopenic
11. Answer: B. NPH is intermediate acting insulin. The symptom; as the brain receives insufficient glucose,
onset of action is 1-2 hours, it peaks in 6-14 hours, and the activity of the CNS becomes depressed.
23. Answer: B. Regular insulin (Humulin R) is a short-acting client is stabilized, subcutaneous insulin is given.
insulin and is administered via IV with an initial dose of Insulin is never given intramuscularly.
0.3 units/kg, followed by 0.2 units/kg 1 hour later, 37. Answer: D. Insulin forces potassium out of the plasma,
followed by 0.2 units/kg every 2 hours until blood back into the cells, causing hypokalemia. Potassium is
glucose becomes <13.9 mmol/L (<250 mg/dL). At this needed to help transport glucose and insulin into the
point, insulin dose should be decreased by half, to 0.1 cells. Calcium, magnesium, and phosphorus aren’t
units/kg every 2 hours, until the resolution of DKA. affected by insulin.
24. Answer: B. The fingertip is preferred for glucose 38. Answer: D. Undiagnosed, untreated DM is one of the
monitoring if hypoglycemia, not hyperglycemia, is most common causes of HHNS.
suspected. 39. Answer: A. Coma and severe acidosis are ushered in
25. Answer: B, C. Metabolic acidosis initiates respiratory with Kussmaul’s respirations (very deep but not
compensation in the form of Kussmaul respirations to labored respirations) and a fruity odor on the breath
counteract the effects of ketone buildup, resulting in a (academia).
lowered PCO2. A fruity odor to the breath (acetone 40. Answer: C. During periods of infection or illness,
breath) occurs when the ketone level is elevated in diabetics may need even more insulin to compensate
ketoacidosis. for increased blood glucose levels.
26. Answer: D. The bicarbonate-carbonic acid buffer 41. Answer: C, D, E. One function of your pancreas is to
system helps maintain the pH of the body fluids; in secrete insulin. The endocrine function of the pancreas
metabolic acidosis there is a decrease in bicarbonate is to secrete insulin. The endocrine, not the exocrine,
because of an increase of metabolic acids. function of the pancreas is to secrete insulin. Insulin is
27. Answer: C. Glucagon, an insulin antagonist produced secreted by the beta, not the alpha, cells of the
by the alpha cells in the islets of langerhans, leads to pancreas. A consequence of diabetes mellitus type 1 is
the conversion of glycogen to glucose in the liver. that without insulin, severe metabolic disturbances,
28. Answer: C. A value of 258mg/dl is above the expected such as ketoacidosis (DKA) will result.
range of 70-105 mg/dl; the nurse should administer 42. Answer: B, C. Regular insulin is the only insulin that can
the insulin as ordered. be given intravenously (IV). The nurse did not use
29. Answer: 1, 3, 2, 4. correct insulin as it was not regular insulin. Contact the
30. Answer: B. Lispro has an immediate onset, a peak of provider to clarify the order, regular insulin is the only
30-90 minutes, and duration of 2-4 hours. insulin that can be given intravenously (IV). The nurse
31. Answer: A. An understanding of the diet is imperative cannot give the insulin subcutaneously when it is
for compliance. A balance of carbohydrates, proteins, ordered to be given intravenously (IV).
and fats usually apportioned over three main meals 43. Answer: D. The client will have moist skin. Moist skin is
and two-between meals snacks needs to be tailored to the sign of hypoglycemia, which the client would
the client’s specific needs, with due regard for activity, experience if he injected himself with insulin and did
diet, and therapy. not eat. Thirst, nausea, and and increased urination
32. Answer: A. Because the client has diabetes, it is are signs of hyperglycemia.
essential that the blood glucose level be determined 44. Answer: C. The 42 year-old client who is 50 pounds
before meals to evaluate the success of control of overweight. Obesity increases the likelihood of
diabetes and the possible need for insulin coverage. developing diabetes mellitus due to over stimulation
33. Answer: C. When a client who has taken an oral of the endocrine system. Exercise is important, but lack
antidiabetic agent vomits, the nurse would monitor of exercise is not as big a risk factor as obesity.
glucose and assess him frequently for signs of Smoking is a serious health concern, but is not a
hypoglycemic. Most of the medication has probably specific risk factor for diabetes. Consuming alcohol is
been absorbed. Therefore, repeating the dose would associated with liver disease but is not as high a risk
further lower glucose levels later in the day. Giving factor for diabetes as obesity.
insulin would also lower the glucose levels, causing 45. Answer: A. Potassium. Insulin causes potassium to
hypoglycemic. The client wouldn’t have hyperglycemia move into the cell and may cause hypokalemia. There
if the glyburide was absorbed. is no need to monitor the sodium, serum amylase, and
34. Answer: B. These are all chronic complications of AST levels.
diabetes. Dizziness, dyspnea on exertion, and coronary 46. Answer: B, C, E. Metformin (Glucophage) reduces
artery disease are symptoms of aortic valve stenosis. insulin resistance, decreases sugar production in the
Fatigue, nausea, vomiting, muscle weakness, and liver, and should be taken with meals for the best
cardiac arrhythmias are symptoms of absorption and effect. It does not stimulate the
hyperparathyroidism. Leg ulcers, cerebral ischemic pancreas to produce more insulin and does not inhibit
events, and pulmonary infarcts are complications of the absorption of carbohydrates.
sickle cell anemia. 47. Answer: A. 1, 2, 3, 4. When a patient with diabetes
35. Answer: B. Insulin lipodystrophy produces fatty masses mellitus type 1 is unresponsive, the the nurse should
at the injection sites, causing unpredictable absorption focus on and treat for hypoglycemia, as this is more
of insulin injected into these sites. likely than hyperglycemia. This is an emergency
36. Answer: D. An IV bolus of insulin is given initially to situation where the nurse must act before calling the
control the hyperglycemia; followed by a continuous physician. Vital signs should be taken after the client is
infusion, titrated to control blood glucose. After the treated for hypoglycemia. Assessment for ABCs should
precede calling a code; there is no information that the 56. Answer: D. All of the above. The principal goals of
client is not breathing. therapy for older persons with diabetes mellitus and
48. Answer: C. Notify the physician and request orders. It poor glycemic control are enhancing quality of life,
is best to notify the client’s physician and request decreasing the chance of complications, improving
orders. The client should not receive the medication self-care through education, and maintaining or
during NPO status unless directed by the physician. improving general health status.
The medications should not be given upon return 57. Answer: A. Physical exercise can slow the progression
unless the physician orders this; the client may still of diabetes mellitus. Physical exercise slows the
need to be NPO. The radiologist in the diagnostic progression of diabetes mellitus, because exercise has
department might give orders, but it would be best to beneficial effects on carbohydrate metabolism and
check with the client’s physician first. insulin sensitivity. Strenuous exercise can cause retinal
49. Answer: B. “Insulin can’t be in a pill because it is damage, and can cause hypoglycemia. Insulin and
destroyed in stomach acid.” Insulin must be injected foods both must be adjusted to allow safe
because it is destroyed in the stomach acid if taken participation in exercise.
orally. Telling he will get used to shots does not 58. Answer: B. Somogyi phenomenon. Somogyi
answer his question and is condescending. Insulin must phenomenon manifests itself with nocturnal
be injected because it is destroyed in stomach acid if hypoglycemia, followed by a marked increase in
taken orally; the onset of action is not the issue here. glucose and increase in ketones.
The nurse should answer the client’s question, not 59. Answer: C. Stimulate the beta cells of the pancreas to
refer him back to the physician. secrete insulin. Sulfonylureas such as glyburide are
50. Answer: D. “I should only use calibrated insulin syringe used only with patients who have some remaining
for the injections.” To ensure the correct insulin dose, pancreatic-beta cell function. These drugs stimulate
a calibrated insulin syringe must be used. Insulin insulin secretion, which reduces liver glucose output
injections should be rotated to the arm and thigh, not and increases cell uptake of glucose, enhancing the
just the abdominal area. There is no need to apply number of and sensitivity of cell receptor sites for
direct pressure over the site following an insulin interaction with insulin.
injection. There is no need to check blood glucose 60. Answer: B. Release insulin evenly throughout the day
immediately prior to the injection. and control basal glucose levels. Glargine (Lantus)
51. Answer: C. The alcohol could predispose you to insulin is designed to release insulin evenly throughout
hypoglycemia. Alcohol can potentiate hypoglycemic, the day and control basal glucose levels.
not hypoglycemic, effects in the client. Alcohol can 61. Answer: D. Pancreatic tumor. The onset of
cause pancreatic disease, but the client’s pancreas is hyperglycemia in the older adult can occur more
not producing any insulin currently. Alcohol can cause slowly. When the older adult reports rapid-onset
liver disease, but the more immediate concern is weight loss, elevated blood glucose levels, and
hypoglycemia. polyphagia, the healthcare provider should consider
52. Answer: A. When the client is eating. The onset action pancreatic tumor.
for the insulin lispro (Humalog) is 10 to 15 minutes so it 62. Answer: D. Perineal itching. Older women might
must be given when the client is eating to prevent complain of perineal itching due to vaginal candidiasis.
hypoglycemia. It must be given when the client is 63. Answer: A, B. When an older person is identified as
eating, not when the meal trays arrive on the floor and high-risk for diabetes, appropriate testing would
not thirty minutes before meals. include FPG and OGTT. A FPG greater than 140 mg/dL
53. Answer: C. Have the client place the insulin vials in the usually indicates diabetes. The OGTT is to determine
refrigerator. Vials not in use should be refrigerated to how the body responds to the ingestion of
preserve drug potency. There is no need to discard the carbohydrates in a meal. HbA1C evaluates long-term
vials. The client should always have additional vials of glucose control. A finger stick glucose three times daily
insulin available. Writing the date of opening on the spot-checks blood glucose levels.
vial is good practice, but does not address the need to 64. Answer: A, B, D. Aging results in reduced ability of beta
refrigerate additional vials. cells to respond with insulin effectively. Overweight
54. Answer: D. Lantus insulin 20U BID. Lantus insulin is with waist/hip ratio increase is part of the metabolic
usually prescribed once-a-day so an order for BID syndrome of DM II. There is an increase in
dosing should be validated with the physician. atherosclerosis with DM due to the metabolic
Humalog insulin can be prescribed for sliding scale syndrome and sedentary lifestyle.
coverage. Regular insulin is administered 30 minutes 65. Answer: C. African-American woman. Age-specific
before meals. Metformin (Glucophage) is often prevalence of diagnosed diabetes mellitus (DM) is
prescribed in divided doses of 1000 mg per day. higher for African-Americans and Hispanics than for
55. Answer: D. “My cells cannot use the insulin my Caucasians. Among those younger than 75, black
pancreas makes.” With type 2 diabetes mellitus, the women had the highest incidence.
pancreas produces insulin, but the cells cannot use it. 66. Answer: D. Hyperglycemic hyperosmolar non-ketotic
Peripheral cells have a decreased, not an increased, coma.Illness, especially with the frail elderly patient
sensitivity to insulin. The beta cells continue to whose appetite is poor, can result in dehydration and
produce insulin with type 2 diabetes. There is a HHNC. Insulin resistance usually is indicated by a daily
decrease, not an increase, in receptor sites with type 2 insulin requirement of 200 units or more. Diabetic
diabetes. ketoacidosis, an acute metabolic condition, usually is
caused by absent or markedly decreased amounts of 78. Answer: B. Higher than 126 mg/dl. Diabetes is
insulin. diagnosed at fasting blood glucose of greater than or
67. Answer: D. Propylthiouracil (PTU). Propylthiouracil equal to 126 mg/dl.
(PTU) initially is given in divided doses, and functions 79. Answer: B. An increase in three areas: thirst, intake of
to block thyroid hormone synthesis. fluids, and hunger. The primary manifestations of
68. Answer: A, B, D. Acetylsalicylic acid (aspirin), diabetes type I are polyuria (increased urine output),
Furosemide (Lasix), and Rifampin (Rifadin) are most polydipsia (increased thirst), polyphagia (increased
likely to cause hypothyroidism. hunger).
69. Answer: C. Ultrasonography. Although thyroid scans 80. Answer: C. Lose a pound a week until weight is in
frequently are done to evaluate the thyroid gland, I normal range for height and exercise 30 minutes daily.
123 is used to destroy overactive thyroid cells such as When Type II diabetics lose weight through diet and
are seen in thyroid cancer. Ultrasonography can be exercise they sometimes have an improvement in
used early in the evaluation process to rule out Graves’ insulin efficiency sufficient to the degree they no
disease, nodular goiter, or other thyroid dysfunction. longer require oral hypoglycemic agents.
70. Answer: C. Hypothermia related to slowed metabolic 81. Answer: A. A client with an HDL cholesterol level of 40
rate. Thyroid hormone deficiency results in reduction mg/dl and a triglyceride level of 300 mg/dl. The seven
in the metabolic rate, resulting in hypothermia, and risk criteria include: greater than 120% of standard
does predispose the older adult to a host of other body weight, Certain races but not including Caucasian,
health-related issues. One quarter of affected elderly delivery of a baby weighing more than 9 pounds or a
experience constipation. diagnosis of gestational diabetes, hypertensive, HDL
71. Answer: A. Try to limit simple sugars to between 10% greater than 35 mg/dl or triglyceride level greater than
and 20% of daily calories. It is recommended that 250 or a triglyceride level of greater than 250 mg/dl,
carbohydrates provide 50% to 60% of the daily and, lastly, impaired glucose tolerance or impaired
calories. Approximately 40% to 50% should be from fasting glucose on prior testing.
complex carbohydrates. The remaining 10% to 20 % of 82. Answer: D. 0.45% normal saline solution. 0.45% normal
carbohydrates could be from simple sugars. Studies saline solution is recommended.
give no evidence that carbohydrates from simple 83. Answer: C. Taking alcohol and/or aspirin with a
sugars are digested and absorbed more rapidly that sulfonylurea drug can cause development of
are complex carbohydrates, and they do not appear to hypoglycemia. Alcohol and/or aspirin taken with a
affect blood sugar control. sulfonylurea can cause development of hypoglycemia.
72. Answer: D. acidosis. When a client’s carbohydrate 84. Answer: B. It has a higher mortality rate than Diabetic
consumption is inadequate, ketones are produced Ketoacidosis. HHNS occurs only in people with Type II
from the breakdown of fat. These ketones lower the Diabetes. It is a medical emergency and has a higher
pH of the blood, potentially causing acidosis that can mortality rate than Diabetic Ketoacidosis. This
lead to a diabetic coma. condition develops very slowly over hours or days.
73. Answer: C. Eating small meals with two or three snacks 85. Answer: A. Pinch the skin up and use a 90 degree
may be more helpful in maintaining blood glucose angle. The best angle for a thin person is 90 degrees
levels than three large meals. Eating small meals with with the skin pinched up. The area is not massaged and
two or three snacks may be more helpful in it is not necessary to warm it.
maintaining blood glucose levels than three large 86. Answer: D. Smoking. Additional risk factors for type 2
meals. diabetes are a family history of diabetes, impaired
74. Answer: A. alcohol use. Alcohol use is one of the most glucose metabolism, history of gestational diabetes,
common risk factors for pancreatitis in the United and race/ethnicity. African-Americans,
States. Hispanics/Latinos, Asian Americans, Native Hawaiians,
75. Answer: A, B, C, D. Gestational diabetes can occur Pacific Islanders, and Native Americans are at greater
between the 16th and 28th week of pregnancy. If not risk of developing diabetes than whites.
responsive to diet and exercise, insulin injections may 87. Answer: A. Below 5.7%. A1c measures the percentage
be necessary. Concentrated sugars should be avoided. of hemoglobin that is glycated and determines average
Weight gain should continue, but not in excessive blood glucose during the 2 to 3 months prior to testing.
amounts. Usually, gestational diabetes disappears Used as a diagnostic tool, A1C levels of 6.5% or higher
after the infant is born. However, diabetes can develop on two tests indicate diabetes. A1C of 6% to 6.5% is
5 to 10 years after the pregnancy. considered prediabetes.
76. Answer: B. <130 mg/dl. The goal for pre-prandial blood 88. Answer: B. Biguanides. Biguanides, such as metformin,
glucose for those with Type 1 diabetes mellitus is <130 lower blood glucose by reducing the amount of glucose
mg/dl. produced by the liver. Sulfonylureas and Meglitinides
77. Answer: B. Unlimited intake of total fat, saturated fat stimulate the beta cells of the pancreas to produce
and cholesterol. The guidelines for Carbohydrate more insulin. Alpha-glucosidase inhibitors block the
Counting as medical nutrition therapy for diabetes breakdown of starches and some sugars, which helps
mellitus includes all of the following EXCEPT option B, to reduce blood glucose levels
unlimited intake of total fat, saturated fat and 89. Answer: C. Give 4 to 6 oz (118 to 177 mL) of orange
cholesterol. juice. Because the client is awake and complaining of
symptoms, the nurse should first give him 15 grams of
carbohydrate to treat hypoglycemia. This could be 4 to
6 oz of fruit juice, five to six hard candies such as 3) Which nursing diagnosis takes highest priority for a client
Lifesavers, or 1 tablespoon of sugar. When a client has with hyperthyroidism?
worsening symptoms of hypoglycemia or is
unconscious, treatment includes 1 mg of glucagon Risk for imbalanced nutrition: More than body requirements
subcutaneously or intramuscularly, or 50 mL of 50% related to thyroid hormone excess
glucose I.V. The nurse may also give two to three Risk for impaired skin integrity related to edema, skin
glucose tablets for a hypoglycemic reaction. fragility, and poor wound healing
90. Answer: B. gives a small continuously dose of regular Body image disturbance related to weight gain and edema
insulin subcutaneously, and the client can self- Imbalanced nutrition: Less than body requirements related to
administer a bolus with an additional dose from the thyroid hormone excess
pump before each meal. An insulin pump provides a 4) A 38 year old woman returns from a subtotal
small continuous dose of regular insulin thryroidectomy for the treatment of hyperthyroidism. Upon
subcutaneously throughout the day and night, and the assessment, the immediate priority that the nurse would
client can self-administer a bolus with an additional include is:
dose from the pump before each meal as needed.
Regular insulin is used in an insulin pump. An external Assess for pain
pump is not attached surgically to the pancreas. Assess for neurological status
91. Answer: C. African-American woman. Age-specific Assess fluid volume status
prevalence of diagnosed diabetes mellitus (DM) is Assess for respiratory distress
higher for African-Americans and Hispanics than for 5) Which of the following nursing assessment is the most
Caucasians. Among those younger than 75, black important in the patient with hyperthyroidism and risk for
women had the highest incidence. thyrotoxic crisis or thyroid storm?
92. Answer: A, B, D. Aging results in reduced ability of beta
cells to respond with insulin effectively. Overweight Intake and output
with waist/hip ratio increase is part of the metabolic Heart sounds
syndrome of DM II. There is an increase in Bowel sounds
atherosclerosis with DM due to the metabolic Vital signs
syndrome and sedentary lifestyle. 6) A client with hyperthyroidism is taking lithium carbonate
93. Answer: A, B. When an older person is identified as to inhibit thyroid hormone release. Which complaint by the
high-risk for diabetes, appropriate testing would client should alert the nurse to a problem with the client’s
include FPG and OGTT. A FPG greater than 126 mg/dL medication?
usually indicates diabetes. The OGTT is to determine
how the body responds to the ingestion of The client complains of blurred vision.
carbohydrates in a meal. HbA1C evaluates long-term The client complains of increased thirst and increased
glucose control. A finger stick glucose three times daily urination.
spot-checks blood glucose levels. The client complains of increased weight gain over the past
94. Answer: C. diaphoresis and trembling. Diaphoresis and year.
trembling indicates hypoglycemia. The client complains of ringing in the ears.
95. Answer: A. confused with cold, clammy skin and pulse 7) Untreated hyperthyroidism during pregnancy may result in
of 110. Confused with cold, clammy skin and pulse of all of the following except:
110 indicate hypoglycemia.
Premature birth and miscarriage
Hyperthyroidism 15 items Low birthweight
1) Which change in vital signs would you instruct a nursing Autism
assistant to report immediately for a patient with Preeclampsia
hyperthyroidism? 8) The physician orders laboratory tests to confirm
hyperthyroidism in a female client with classic signs and
Increased and rapid heart rate symptoms of this disorder. Which test result would confirm
Decrease systolic blood pressure the diagnosis?
Increased respiratory rate
Decreased oral temperature No increase in the thyroid-stimulating hormone (TSH) level
2) For the patient with hyperthyroidism, what intervention after 30 minutes during the TSH stimulation test
should you delegate to the experienced certified nursing A decreased TSH level
assistant? An increase in the TSH level after 30 minutes during the TSH
stimulation test
Instruct the patient to report palpitations, dyspnea, vertigo, Below-normal levels of serum triiodothyronine (T3) and
pr chest pain. serum thyroxine (T4) as detected by radioimmunoassay
Check the apical pulse, blood pressure, and temperature 9) A client is being returned after subtotal thyroidectomy.
every 4 hours. Which piece of equipment is most important for the nurse to
Draw blood for thyroid-stimulating hormone, T3, and T4 keep at the client’s bedside?
levels.
Explain the side effects of propylthiouracil (PTU) to the Orange juice and hard candy
patient. Tracheostomy set
Cardiac monitor and oxygen tank 2. B. Check the apical pulse, blood pressure, and
Indwelling catheter tray temperature every 4 hours. Monitoring and recording
10) Which medication will the nurse have available for vital signs are within the education scope of nursing
emergency treatment of tetany in the client who has had assistants. An experienced nursing assistant should
thyroidectomy? have been taught how to monitor the apical pulse.
However, the nurse should observe the nursing
Calcium chloride assistant to be sure that she has mastered this skill.
Potassium chloride Instructing and teaching patients, as well as
Magnesium chloride performing venipuncture for laboratory samples, are
Sodium bicarbonate more suited to the educational scope of licensed
11) Matilda, with hyperthyroidism is to receive Lugol’s iodine nurses. In some facilities, an experienced nursing
solution before a subtotal thyroidectomy is performed. The assistant may perform venipuncture, but only after
nurse is aware that this medication is given to: special training. Focus: Delegation/supervision
3. D. Imbalanced nutrition: Less than body requirements
Decrease the total basal metabolic rate. related to thyroid hormone excess . In the client with
Maintain the function of the parathyroid glands. hyperthyroidism, excessive thyroid hormone
Block the formation of thyroxine by the thyroid gland. production leads to hypermetabolism and increased
Decrease the size and vascularity of the thyroid gland. nutrient metabolism. These conditions may result in a
12) Which nursing diagnosis takes highest priority for a negative nitrogen balance, increased protein synthesis
female client with hyperthyroidism? and breakdown, decreased glucose tolerance, and fat
mobilization and depletion. This puts the client at risk
Risk for imbalanced nutrition: More than body requirements for marked nutrient and calorie deficiency, making
related to thyroid hormone excess Imbalanced nutrition: Less than body requirements the
Risk for impaired skin integrity related to edema, skin most important nursing diagnosis.
fragility, and poor wound healing 4. D. Assess for respiratory distress . Though fluid volume
Body image disturbance related to weight gain and edema status, neurological status and pain are all important
Imbalanced nutrition: Less than body requirements related to assessment, the immediate priority for postoperative
thyroid hormone excess is the airway management. Respiratory distress may
13) Ms.Clark has hyperthyroidism and is scheduled for a result from hemorrhage, edema, laryngeal damage or
thyroidectomy. The physician has ordered Lugol’s solution for tetany.
the client. The nurse understands that the primary reason for 5. D. Vital signs
giving Lugol’s solution preoperatively is to: 6. B. The client complains of increased thirst and
increased urination. Increased thirst and increased
decrease the risk of agranulocytosis postoperatively. urination are signs of lithium toxicity.
prevent tetany while the client is under general anesthesia. 7. C. Autism . In addition to the above-mentioned
reduce the size and vascularity of the thyroid and prevent complications of uncontrolled hyperthyroidism in
hemorrhage. pregnancy, expectant mothers may suffer congestive
potentiate the effect of the other preoperative medication so heart failure and thyroid storm, which is life-
less medicine can be given while the client is under threatening thyrotoxicosis with symptoms that include
anesthesia. agitation, confusion, tachycardia, shaking, sweating,
14) What are the functions of T3 and T4? diarrhea, fever, and restlessness.
8. A. No increase in the thyroid-stimulating hormone
Retention of salt and water (TSH) level after 30 minutes during the TSH stimulation
Maintenance of blood sugar test . In the TSH test, failure of the TSH level to rise
Maintenance of blood pressure after 30 minutes confirms hyperthyroidism. A
Regulation of energy production decreased TSH level indicates a pituitary deficiency of
15) A patient is admitted to the medical unit with possible this hormone. Below-normal levels of T3 and T4, as
Graves’ disease (hyperthyroidism). Which assessment finding detected by radioimmunoassay, signal
supports this diagnosis? hypothyroidism. A below-normal T4 level also occurs
in malnutrition and liver disease and may result from
Periorbital edema administration of phenytoin and certain other drugs.
Bradycardia 9. B. Tracheostomy set . After subtotal thyroidectomy,
Exophthalmos swelling of the surgical site ( the tracheal area) may
Hoarse voice obstruct the airway. Therefore tracheostomy set
should be at the bedside in case of respiratory
Answers and Rationales emergency.
1. A. Increased and rapid heart rate . The cardiac 10. B. Potassium chloride
problems associated with hyperthyroidism include 11. D. Decrease the size and vascularity of the thyroid
tachycardia, increased systolic blood pressure, and gland. Lugol’s solution provides iodine, which aids in
decreased diastolic blood pressure. Patients with decreasing the vascularity of the thyroid gland, which
hyperthyroidism also may have increased body limits the risk of hemorrhage when surgery is
temperature related to increased metabolic rate. performed.
Focus: Delegation/supervision
12. D. Imbalanced nutrition: Less than body requirements pituitary
related to thyroid hormone excess . In the client with intestinal
hyperthyroidism, excessive thyroid hormone none of the above
production leads to hypermetabolism and increased 5) A nurse gets back results on a pt who may have
nutrient metabolism. These conditions may result in a hyperthyroidism. What lab values might the nurse expect?
negative nitrogen balance, increased protein synthesis
and breakdown, decreased glucose tolerance, and fat decreased T3 and Increased T4
mobilization and depletion. This puts the client at risk Decreased TSH and Increased T3 and T4
for marked nutrient and calorie deficiency, making Increased TSH and decreased T3 and T4
Imbalanced nutrition: Less than body requirements the Increased T3 and Decreased T4
most important nursing diagnosis. 6) The physician has discussed the need for medication with
13. C. reduce the size and vascularity of the thyroid and the parents of an infant with congenital hypothyroidism. The
prevent hemorrhage. Doses of over 30 mg/day may nurse can reinforce the physician’s teaching by telling the
increase the risk of agranulocytosis. Lugol’s solution parents that:
does not act to prevent tetany. Calcium is used to treat
tetany. The client may receive iodine solution (Lugol’s The medication will be needed only during times of rapid
solution) for 10 to 14 days before surgery to decrease growth.
vascularity of the thyroid and thus prevent excess The medication will be needed throughout the child’s
bleeding. Lugol’s solution does not potentiate any lifetime.
other preoperative medication. The medication schedule can be arranged to allow for drug
14. D. Regulation of energy production holidays.
15. C. Exophthalmos . Exophthalmos (abnormal protrusion The medication is given one time daily every other day.
of the eye) is characteristic of patients with 7) A client has been newly diagnosed with hypothyroidism
hyperthyroidism due to Graves’ disease. Periorbital and will take levothyroxine (Synthroid) 50 mcg/day by
edema, bradycardia, and hoarse voice are all mouth. As part of the teaching plan, the nurse emphasizes
characteristics of patients with hypothyroidism. Focus: that this medication:
Prioritization
Should be taken in the morning
Hypothyroidism 30 items May decrease the client’s energy level
1) A 32-year-old mother of three is brought to the clinic. Her Must be stored in a dark container
pulse is 52, there is a weight gain of 30 pounds in 4 months, Will decrease the client’s heart rate
and the client is wearing two sweaters. The client is 8) Iodine’s main function in therapy is to:
diagnosed with hypothyroidism. Which of the following
nursing diagnoses is of highest priority? Rapidly inhibits the synthesis/release of T3 & T4
Blocks conversion of T4 to T3
Impaired physical mobility related to decreased endurance Damages or destroys thyroid tissue
Hypothermia r/t decreased metabolic rate Decreases HR, BP, CO2, and O2 requirements
Disturbed thought processes r/t interstitial edema 9) A nurse gets back results on a pt who may have
Decreased cardiac output r/t bradycardia hypothyroidism. What lab values might the nurse expect?
2) Despite the availability of a wide array of thyroid hormone
products, it is clear that ___________ is the treatment of decreased T3 and Increased T4
choice for almost all patients with hypothyroidism. increased TSH and Increased T3 and T4
Increased TSH and decreased T3 and T4
Liothyronine (Cytomel) Increased T3 and Decreased T4
Liotrix (Thryolar) 10) On discharge, the nurse teaches the patient to observe
Synthetic levothyroxine (LT4) for signs of surgically induced hypothyroidism. The nurse
Desiccared Thyroid (Armour, etc) would know that the patient understands the teaching when
None of the above she states she should notify the MD if she develops:
3) High TSH but low FT4 suggests…Hint: TSH and FT4 levels
going in opposite directions is usually what we expect to see, Intolerance to heat
generally indicating a problem on the level of the thyroid. Dry skin and fatigue
Progressive weight gain
Primary hyperthyroidism Insomnia and excitability
Primary hypothyroidism 11) Which of the following potentially serious complications
Pituitary hyperthyroidism could occur with therapy for hypothyroidism?
Pituitary hypothyroidism
Hypothalamus hypothyroidism Acute hemolytic reaction.
4) A radioactive iodide uptake test helps determine intrinsic Angina or cardiac arrhythmia.
thyroid function. A thyroid scan can help distinguish etiology Retinopathy.
(such as cancer). A TRH stimulation test helps determine Thrombocytopenia.
thyroid and ________ function. 12) Some causes of hypothyroidism include iatrogenic causes,
drugs (such as amiodarone and lithium), and this (also known
liver as autoimmune thyroiditis)….
Damages or destroys thyroid tissue
Grave’s Decreases HR, BP, CO2, and O2 requirements
Hashimoto’s 21) Michael comes into the ED. His HR is 150 and has a fever
Silent thyroiditis of 104.5. He is confused and anxious. What could be
Hypothalamic disease Michael’s problem?
Pituitary disease
13) A client with hypothyroidism asks the nurse if she will still Thyroid infection.
need to take thyroid medication during the pregnancy. The Thyroid Fire
nurse’s response is based on the knowledge that: Thyroid Storm
Thyroid Shut Down.
There is no need to take thyroid medication because the 22) After undergoing a subtotal thyroidectomy, a female
fetus’s thyroid produces a thyroid-stimulating hormone. client develops hypothyroidism. Dr. Smith prescribes
Regulation of thyroid medication is more difficult because the levothyroxine (Levothroid), 25 mcg P.O. daily. For which
thyroid gland increases in size during pregnancy. condition is levothyroxine the preferred agent?
It is more difficult to maintain thyroid regulation during
pregnancy due to a slowing of metabolism. Primary hypothyroidism
Fetal growth is arrested if thyroid medication is continued Graves’ disease
during pregnancy. Thyrotoxicosis
14) A nurse understands calcitonin and parathormone when Euthyroidism
the nurse states: 23) A client is admitted with a diagnosis of hypothyroidism.
An initial assessment of the client would reveal:
They work separately on calcium balance.
They work together for blood clotting. Slow pulse rate, weight loss, diarrhea, and cardiac failure
They work together on bone growth. Weight gain, lethargy, slowed speech, and decreased
Calcitonin works on cellular function while parathormone respiratory rate
works on neuromuscular function. Rapid pulse, constipation, and bulging eyes
15) Mrs. Gilmour is complaining of being hot all the time, Decreased body temperature, weight loss, and increased
sweating, being thirsty, and losing hair. What could her respirations
symptoms indicate? 24) A 58 year old woman has newly diagnosed with
hypothyroidism. The nurse is aware that the signs and
Hypothyroidsim symptoms of hypothyroidism include:
Hyperthyroidism
Graves disease Diarrhea
16) Thyrotoxicosis is understood when the nurse states: Vomiting
Tachycardia
Hypermetabolism from decrease in T3 and T4. Weight gain
Hypometabolism from decrease in T3 and T4. 25) A pt is given radioactive iodine for a thyroid scan. How
Hypermetabolism from increase in T3 and T4. often are the tests taken?
Hypometabolism from increase in T3 and T4.
17) T3 and T4 are produced in which type of cell? one hour after ingestion
4-8-24 hours
Follicle 2-4-24 hours
Squamous 2-6-24 hours
Epithelial 26) T3 and T4 are similar, but different produced in equal
Glandular amounts?
18) A newborn has been diagnosed with hypothyroidism. In
discussing the condition and treatment with the family, the True
nurse should emphasize False
27) A client with hypothyroidism frequently complains of
They can expect the child will be mentally retarded feeling cold. The nurse should tell the client that she will be
Administration of thyroid hormone will prevent problems more comfortable if she:
This rare problem is always hereditary
Physical growth/development will be delayed Uses an electric blanket at night
19) What does tyrosine combine with to make T3 and T4? Dresses in extra layers of clothing
Applies a heating pad to her feet
Serum K Takes a hot bath morning and evening
Serum Na 28) A female client with hypothyroidism (myxedema) is
Serum Iodine receiving levothyroxine (Synthroid), 25 mcg P.O. daily. Which
Serum Phosphoruse finding should nurse Hans recognize as an adverse drug
20) Radioactive Iodine’s main function in therapy is to: effect?

Rapidly inhibits the synthesis/release of T3 & T4 Dysuria


Blocks conversion of T4 to T3 Leg cramps
Tachycardia 13. B. Regulation of thyroid medication is more difficult
Blurred vision because the thyroid gland increases in size during
29) Nurse Oliver should expect a client with hypothyroidism pregnancy. During pregnancy, the thyroid gland triples
to report which health concerns? in size. This makes it more difficult to regulate thyroid
medication.
Increased appetite and weight loss 14. B. They work together for blood clotting. , C. They work
Puffiness of the face and hands together on bone growth.
Nervousness and tremors 15. B. Hyperthyroidism
Thyroid gland swelling 16. C. Hypermetabolism from increase in T3 and T4.
30) An incoherent female client with a history of 17. A. Follicle
hypothyroidism is brought to the emergency department by 18. B. Administration of thyroid hormone will prevent
the rescue squad. Physical and laboratory findings reveal problems . Early identification and continued
hypothermia, hypoventilation, respiratory acidosis, treatment with hormone replacement corrects this
bradycardia, hypotension, and nonpitting edema of the face condition.
and pretibial area. Knowing that these findings suggest 19. C. Serum Iodine
severe hypothyroidism, nurse Libby prepares to take 20. C. Damages or destroys thyroid tissue
emergency action to prevent the potential complication of: 21. C. Thyroid Storm
22. A. Primary hypothyroidism. Levothyroxine is the
Thyroid storm. preferred agent to treat primary hypothyroidism and
Cretinism. cretinism, although it also may be used to treat
myxedema coma. secondary hypothyroidism. It is contraindicated in
Hashimoto’s thyroiditis. Graves’ disease and thyrotoxicosis because these
31) A 3-month-old client is in the pediatric unit. During conditions are forms of hyperthyroidism.
assessment, the nurse is suspecting that the baby may have Euthyroidism, a term used to describe normal thyroid
hypothyroidism when mother states that her baby does not: function, wouldn’t require any thyroid preparation.
23. B. Weight gain, lethargy, slowed speech, and
Sit up. decreased respiratory rate . Symptoms of
Pick up and hold a rattle. hypothyroidism include weight gain, lethargy, slow
Roll over. speech, and decreased respirations.
Hold the head up. 24. D. Weight gain . Typical signs of hypothyroidism
includes weight gain, fatigue, decreased energy,
Answers and Rationales apathy, brittle nails, dry skin, cold intolerance,
1. D. Decreased cardiac output r/t bradycardia . The constipation and numbness.
decrease in pulse can affect the cardiac output and 25. D. 2-6-24 hours
lead to shock, which would take precedence over the 26. B. False. T3 and T4 are similar, but they are produced
other choices in different amounts.
2. C. Synthetic levothyroxine (LT4) 27. B. Dresses in extra layers of clothing. Dressing in layers
3. B. Primary hypothyroidism and using extra covering will help decrease the feeling
4. B. pituitary of being cold that is experienced by the client with
5. B. Decreased TSH and Increased T3 and T4 hypothyroidism. Decreased sensation and decreased
6. B. The medication will be needed throughout the alertness are common in the client with
child’s lifetime. The medication will be needed hypothyroidism; therefore, the use of electric blankets
throughout the child’s lifetime. Answers A, C, and D and heating pads can result in burns,
contain inaccurate statements; therefore, they are 28. C. Tachycardia . Levothyroxine, a synthetic thyroid
incorrect. hormone, is given to a client with hypothyroidism to
7. A. Should be taken in the morning . Should be taken in simulate the effects of thyroxine. Adverse effects of
the morning Thyroid supplement should be taken in this agent include tachycardia. The other options
the morning to minimize the side effects of insomnia aren’t associated with levothyroxine.
8. A. Rapidly inhibits the synthesis/release of T3 & T4 29. B. Puffiness of the face and hands . Hypothyroidism
9. C. Increased TSH and decreased T3 and T4 (myxedema) causes facial puffiness, extremity edema,
10. C. Progressive weight gain . Hypothyroidism, a and weight gain. Signs and symptoms of
decrease in thyroid hormone production, is hyperthyroidism (Graves’ disease) include an increased
characterized by hypometabolism that manifests itself appetite, weight loss, nervousness, tremors, and
with weight gain thyroid gland enlargement (goiter).
11. B. Angina or cardiac arrhythmia. Precipitation of 30. C. myxedema coma. Severe hypothyroidism may result
angina or cardiac arrhythmia is a potentially serious in myxedema coma, in which a drastic drop in the
complication of hypothyroidism treatment. Acute metabolic rate causes decreased vital signs,
hemolytic reaction is a complication of blood hypoventilation (possibly leading to respiratory
transfusions. Retinopathy typically is a complication of acidosis), and nonpitting edema. Thyroid storm is an
diabetes mellitus. Thrombocytopenia doesn’t result acute complication of hyperthyroidism. Cretinism is a
from treating hypothyroidism. form of hypothyroidism that occurs in infants.
12. B. Hashimoto’s Hashimoto’s thyroiditis is a common chronic
inflammatory disease of the thyroid gland in which
autoimmune factors play a prominent role.
31. D. Hold the head up. Development normally proceeds
cephalocaudally; so the first major developmental
milestone that the infant achieves is the ability to hold
the head up within the first 8-12 weeks of life. In
hypothyroidism, the infant’s muscle tone would be
poor and the infant would not be able to achieve this
milestone.

You might also like